Missed MBE Questions Flashcards

Missed questions from the Qbank and Strategies & Tactics

You may prefer our related Brainscape-certified flashcards:
1
Q

State the rule triggered by the question

A plane crashed in State C, killing all 120 passengers on board. The deceased passengers were domiciled in States A, B, and C. The plane was owned by an airline incorporated in State A, headquartered in State B, and licensed to do business in State C. The plane’s manufacturer is incorporated, headquartered, and licensed to do business in State C.

The estates of the deceased passengers brought an action against the airline and the manufacturer in a State B federal court. The estates assert wrongful death claims on behalf of the deceased passengers to recover $120 million in damages. The airline and the manufacturer have moved to dismiss for lack of subject-matter jurisdiction.

Should the court grant the defendants’ motion?

A

A federal court has original subject-matter jurisdiction over an action in which minimal diversity of citizenship exists when the requirements of the Multiparty, Multiforum Trial Jurisdiction Act (MMTJA) are satisfied.

How well did you know this?
1
Not at all
2
3
4
5
Perfectly
2
Q
A

Contracts must be supported by consideration—ie, a bargained-for exchange of promises or performance—which requires each party to (1) perform or promise to perform an act that is not legally required or (2) refrain or promise to refrain from performing some legally permissible act.

How well did you know this?
1
Not at all
2
3
4
5
Perfectly
3
Q

State the rule triggered by the question

A private high school was in the market for new desks and chairs for its classrooms. It had inquired into the cost of acquiring 1,000 new desks and chairs from a particular vendor. On June 15, the vendor sent a signed letter to the private high school offering to sell 1,000 desks and chairs for $30,000. The letter stated that the private high school’s acceptance would be effective only if the vendor actually received it by June 25. On June 23, the private high school mailed its signed, written acceptance of the vendor’s offer. On June 24, the vendor, after concluding that the price it originally requested was too low, directly notified the private high school that it was revoking its offer. The vendor received the private high school’s acceptance on June 26.

Was an enforceable contract between the private high school and the vendor formed?

A

The mailbox rule does not apply if it is inconsistent with the offer’s terms, such as when the offer states that acceptance is effective only upon receipt.

How well did you know this?
1
Not at all
2
3
4
5
Perfectly
4
Q

State the legal rule or principle triggered by the question

A plaintiff brought a diversity action against a defendant in a federal court in State A to quiet title to a farm. The plaintiff attempted to serve the defendant with process through certified mail, but the letter was returned to the plaintiff marked “unclaimed.” The plaintiff attempted service through certified mail a second time, but the letter was returned to the plaintiff in the same fashion. The plaintiff did not attempt to contact the defendant again. At trial, the defendant did not appear and defend. As a result, the court entered a default judgment in the plaintiff’s favor.

Two months later, the defendant discovered the plaintiff on the farm and learned about the default judgment. The defendant immediately moved for relief from the default judgment in the federal court in State A on the ground that the plaintiff failed to provide him with adequate notice of the suit.

State A’s rules of procedure permit service of process by certified or ordinary mail.

Will the federal court likely grant the defendant’s motion for relief?

A

Notice, as required by due process, means that a defendant must be reasonably apprised of the pending suit and afforded an opportunity to present objections. So if the plaintiff knows the defendant did not receive notice through service of process, then the plaintiff must take additional reasonable steps to provide notice.

Notice the plaintiff did not take additional reasonable steps to provide notice

How well did you know this?
1
Not at all
2
3
4
5
Perfectly
5
Q

A retiree was known to walk her beloved dog around the neighborhood daily. One evening, she forgot to shut her gate, and her dog escaped. The retiree placed posters around the neighborhood stating, “Lost dog! $500 reward. No questions asked.”

The dog had wandered to a nearby store, and the store owner kept the dog and fed it for a few days. A customer saw the dog and told the store owner that it looked like the retiree’s dog. The store owner took the dog to the retiree’s home, and the dog was happily reunited with its owner. The store owner had not seen the posters and did not ask for the reward.

On the way back to the store, the store owner saw one of the posters and called the retiree to inquire about the reward. The retiree refused to pay the reward.

What type of offer did retiree create?

Who could accept the offer?

What legal rule is triggered?

A

The reward is a unilateral offer to contract, acceptance is valid only through full performance.

Since store owner didn’t know about the offer he couldn’t accept it. Therefore, there was no mutual assent, no contract, and no breach.

Rule: Offers to form unilateral or bilateral contracts must be reasonably communicated to the offeree, and the offeree must be aware of the offer to accept it. Otherwise, there can be no mutual assent and therefore no contract.

How well did you know this?
1
Not at all
2
3
4
5
Perfectly
6
Q

A woman was driving home from a crowded shopping mall when a passenger van ran through a stoplight at great speed and hit the woman’s car. A physician who witnessed the incident pulled over to the woman’s car and saw that her shoulder was seriously injured. The physician told the woman he was a licensed surgeon before helping her out of the car. The woman was delirious and too shaken to speak as the physician removed several glass fragments from her shoulder and applied temporary sutures to close the wound. When the physician finished, he took the woman’s contact information and promised to check on her condition later.

If the physician subsequently sends the woman a bill for his services, will she be obligated to pay it?

A

Yes!

  • Courts will construct an implied-in-law **(“quasi”) contract **where the plaintiff has conferred a measurable benefit on the defendant without gratuitous intent and it would be unfair to let the defendant retain the benefit without compensating the plaintiff.
  • Unfair retention of a benefit arises when (1) the defendant had an opportunity to decline the benefit but knowingly accepted it or (2) the plaintiff had a reasonable excuse for not giving the defendant such opportunity—often because of an emergency. This allows the plaintiff in a quasi-contract action to recover restitutionary damages equal to the reasonable value of the benefit conferred.
How well did you know this?
1
Not at all
2
3
4
5
Perfectly
7
Q

Can a promise to surrender a claim of defense constitute adequate consideration?

If so what are the requirements?

A

Yes!

A promise to surrender a claim or defense constitutes consideration for a settlement agreement so long as (1) the claim or defense is valid or subject to a good-faith dispute or (2) the surrendering party believes that the claim or defense may be valid.

How well did you know this?
1
Not at all
2
3
4
5
Perfectly
8
Q

What legal principle is triggered by the question?

A shop owner faxed the following signed message to his long-time widget supplier: “Urgently need blue widgets. Ship immediately three gross at your current list price of $600.” Upon receipt of the fax, the supplier shipped three gross of red widgets to the shop owner and faxed to him the following message: “Temporarily out of blue. In case red will help, am shipping three gross at the same price. Hope you can use them.”

Upon the shop owner’s timely receipt of both the shipment and the supplier’s fax, which of the following best describes the rights and duties of the shop owner and the supplier?

A

Under the UCC, a seller can accept an offer to buy goods for prompt or immediate shipment by promising to ship or actually shipping conforming goods. However, shipping nonconforming goods serves as both a rejection and counteroffer if the seller notifies the buyer that the shipment was merely an accommodation.

The shop owner may accept the shipment, in which case he must pay the supplier the list price, or he may reject the shipment and he has no further rights against the supplier.

How well did you know this?
1
Not at all
2
3
4
5
Perfectly
9
Q

Termination of offer before acceptance

A

Once an offer has been made, a binding contract will be formed if the offer is accepted before it terminates. Offers can be terminated by revocation, rejection, lapse, or operation of law (see table above). An offer terminates by operation of law when, for example, the subject matter of the offer is destroyed.

How well did you know this?
1
Not at all
2
3
4
5
Perfectly
10
Q

Termination of offer before acceptance

A

Offeror’s revocation

Offeror communicates revocation directly to offeree

Offeree learns information from reliable source that reasonably indicates offer was revoked (eg, house sold to another buyer)

Offeree’s rejection

Offeree communicates rejection directly to offeror

Offeree’s counteroffer serves as rejection & new offer*

Lapse

Time period specified in offer expires

After reasonable time if no time period specified in offer

By law
Either party dies or is adjudicated insane

Subject matter of offer is destroyed or becomes illegal

How well did you know this?
1
Not at all
2
3
4
5
Perfectly
11
Q

What is the function of a merger clause under the UCC?

A

The UCC, which applies to contracts for the sale of goods (e.g., comic books), presumes that a contract is partially integrated. However, that presumption goes away when the writing contains a merger clause—i.e., a clause that declares the written contract to be the complete and final agreement between the parties. The written contract will instead be deemed completely integrated.

A written contract is completely integrated if it contains a merger clause—i.e., a clause that declares the written contract to be the complete and final agreement between the parties. As a result, the parol evidence rule bars the admission of prior or contemporaneous agreements that modify or contradict the terms of the writing.

How well did you know this?
1
Not at all
2
3
4
5
Perfectly
12
Q
A

A written contract is completely integrated if it contains a merger clause—i.e., a clause that declares the written contract to be the complete and final agreement between the parties. As a result, the parol evidence rule bars the admission of prior or contemporaneous agreements that modify or contradict the terms of the writing.

How well did you know this?
1
Not at all
2
3
4
5
Perfectly
13
Q

under the UCC When is performance due on an installment contract?

A

Under the UCC, an installment contract is defined as a contract in which the goods are to be delivered in multiple shipments, and each shipment is to be separately accepted by the buyer. Payment by the buyer is due upon each delivery unless the price cannot be apportioned.

How well did you know this?
1
Not at all
2
3
4
5
Perfectly
14
Q

What happens when a party’s performance of a condition precedent is prevented?

A

A condition precedent is an explicit contract term requiring a future event to occur before a party becomes obligated to perform. However, under the doctrine of prevention, a condition’s nonoccurrence is excused when the party whose duty to perform is subject to the condition wrongfully prevents or interferes with the occurrence of that condition.

How well did you know this?
1
Not at all
2
3
4
5
Perfectly
15
Q

How can an offer be revived?

A

An offer cannot be accepted after it terminates (e.g., is rejected by the offeree). But the offer can be revived if the offeror conveys that it is still open, which creates a renewed opportunity for the offeree to accept.

How well did you know this?
1
Not at all
2
3
4
5
Perfectly
16
Q

What is an exception to the parol evidence rule when it comes to raising defenses against the formation of a contract? Name one specific defense and describe it.

A

the parol evidence rule does not apply to evidence offered to raise a defense to contract formation. Misrepresentation is one such defense, which arises when a contracting party made an untrue assertion of fact.

How well did you know this?
1
Not at all
2
3
4
5
Perfectly
17
Q
A

To form a contract, an offer must be accepted before it terminates. An offer will terminate if, for example, the offeree rejects it by clearly conveying that he/she does not intend to accept the offer OR by making a counteroffer. Once the offer has terminated, it cannot be accepted. But the offer can be revived if the offeror conveys that it is still open. This creates a renewed opportunity for acceptance by the offeree. If the offeree accepts the revived offer, then a binding contract is formed.

How well did you know this?
1
Not at all
2
3
4
5
Perfectly
18
Q

What are the exceptions to the parol evidence rule?

A

Evidence of prior or contemporaneous oral or written agreement is admissible to establish:

  1. whether writing is integrated and, if so, completely or partially
  2. meaning of ambiguous term
  3. defense to formation or enforcement (eg, fraud, duress, mistake)
  4. ground for granting or denying remedy (eg, rescission, reformation)
  5. subsequent contract modifications
  6. condition precedent to effectiveness

Remember: The parol evidence rule does not apply to evidence offered to raise a defense to contract formation.

How well did you know this?
1
Not at all
2
3
4
5
Perfectly
19
Q

Discuss contract formation under the common law vs. the UCC

A

Contract formation under the common law requires an offer with definite terms and an acceptance with knowledge of that offer. But these requirements are relaxed by the UCC, which governs contracts for the sale of goods (e.g., a ring). Under the UCC, a contract is formed if the parties intended to contract and there is a reasonably certain basis for giving a remedy. The contract may be made in any manner sufficient to show agreement—even if the moment of its making is undetermined.

How well did you know this?
1
Not at all
2
3
4
5
Perfectly
20
Q

What are a buyer’s duty of good faith & fair dealing under requirements contract?

A

Article 2 of the Uniform Commercial Code (UCC) governs contracts for the sale of goods. Under the UCC, a requirements contract is a contract for the sale of as many goods as the buyer requires during a specified period. This creates an exclusive agreement between the buyer and the seller. As a result, the duty of good faith and fair dealing implied in every contract requires the buyer to purchase goods from the seller only. A failure to do so violates that duty and is a breach of contract.

How well did you know this?
1
Not at all
2
3
4
5
Perfectly
21
Q
A
How well did you know this?
1
Not at all
2
3
4
5
Perfectly
22
Q

Auctions

A

During a reserve auction, the auctioneer may withdraw goods from auction prior to completion of the sale (e.g., before the auctioneer’s hammer falls). At a no-reserve auction, goods generally cannot be withdrawn after the auctioneer calls for bids.

How well did you know this?
1
Not at all
2
3
4
5
Perfectly
23
Q

accord and satisfaction

A

If a debt is disputed in good faith, then the debtor can offer to satisfy the debt by giving the creditor a check with a conspicuous “payment in full” notation. But if the debt is certain and undisputed, then it cannot be satisfied by a check for a lesser amount—even if the creditor cashes the check.

How well did you know this?
1
Not at all
2
3
4
5
Perfectly
24
Q

Beneficiaries to a contract

A

Intended beneficiaries receive a direct benefit from a contract because the contracting parties so intended, while incidental beneficiaries receive an indirect benefit from a contract even though there was no contractual intent to benefit them. Only intended (not incidental) beneficiaries can sue to enforce the contract.

How well did you know this?
1
Not at all
2
3
4
5
Perfectly
25
Q

Ways to discharge contractual obligations

Mnemonic: FIRM SCAN

A

Full performance of contractual obligations
Impossibility, impracticability, or frustration of purpose
Release (in writing only)
Mutual rescission
Substituted contract
Contract or covenant not to sue
Accord & satisfaction
Novation

How well did you know this?
1
Not at all
2
3
4
5
Perfectly
26
Q

Does an accord require consideration to be valid?

Remember the question where tutoring services were rendered. Mother offered a $500 bonus if the student received a B or better. The bike was offered instead of the $500 under the original contract because mother didn’t have the cash to pay.

A

Yes, an accord requires consideration to be valid. That consideration can be worth less than what was agreed to in the original contract only if (1) there is a good-faith dispute as to the amount owed or (2) the new consideration is of a different type than what was owed under the original contract.

The tutor agreed to accept the bike (accord) and the mother delivered the bike to the tutor (satisfaction), the original contract was discharged. Therefore, the tutor cannot recover $200.

How well did you know this?
1
Not at all
2
3
4
5
Perfectly
27
Q

What is an accord and how is it created?

A

One method of discharging contractual obligations is by an accord (new contract) and satisfaction (performance of the accord).

An accord is created when the parties to a contract agree to give and accept something different in satisfaction of their existing contractual obligations.

Since an accord is a new contract, it must be supported by consideration. Where the new consideration is worth less than what was agreed to in the original contract, it will be sufficient only if:

  • there is a good-faith dispute as to the amount owed (Choice D) or
  • the new consideration is of a different type than what was owed under the original contract (e.g., goods in lieu of cash).
How well did you know this?
1
Not at all
2
3
4
5
Perfectly
28
Q

Give an example of a promise that is enforceable without consideration.

A

A contract must generally be supported by consideration to be enforceable. However, there are circumstances in which a promise is enforceable without consideration.

For example, a new promise to pay a debt after the statute of limitations has run is enforceable without any new consideration.

When the new promise is an express promise, most jurisdictions require that the new promise be in **writing and signed by the debtor **to be enforceable.

How well did you know this?
1
Not at all
2
3
4
5
Perfectly
29
Q
A

Expectation damages are computed as loss in value + other loss − costs avoided − loss avoided. For late delivery in real estate contracts, loss in value is measured by the fair market rental value of the property for the time that the buyer was denied possession.

How well did you know this?
1
Not at all
2
3
4
5
Perfectly
30
Q

Is the closing date an essential term in real estate contracts?

A

A closing date is not an essential term of a real estate contract. Therefore, a seller’s performance is typically due at or within a reasonable time after the closing date—unless the real estate contract contains a “time is of the essence” clause (not seen here). Accordingly, the seller’s failure to perform by the closing date is not a material breach that excuses the buyer’s duty to perform—but it is still a breach. The buyer can therefore recover damages—even if the seller acted in good faith

How well did you know this?
1
Not at all
2
3
4
5
Perfectly
31
Q

A mother owned a vacation cabin, but as she no longer visited it, she decided to convey the cabin to her daughter. The mother executed a valid, written deed, and she promptly and properly recorded it. The mother did not tell her daughter that she intended to give the cabin to the daughter because the mother wanted to surprise the daughter with this gift at an upcoming family reunion.

Prior to the reunion, the daughter died suddenly. In her will, the daughter left her entire estate to her best friend. The mother, not wanting the cabin to go to someone who was not a family member, brought an action to set aside the conveyance to the best friend.

Who will be likely to prevail in this action?
Quote the legal principle implicated…

A

The best friend, because the mother recorded the deed conveying the cabin to her daughter.

A transfer by deed is effective when the deed is delivered by the grantor and accepted by the grantee. Delivery is presumed when the deed has been recorded, and acceptance is presumed if the transfer benefits the grantee.

How well did you know this?
1
Not at all
2
3
4
5
Perfectly
32
Q

Upon her death, the owner of six acres of undeveloped land devised her property to her brother and sister as “joint tenants with the right of survivorship,” with a one-third interest in the land allocated to the owner’s brother and a two-thirds interest in the land allocated to the owner’s sister. Neither the brother nor the sister transferred their interest in the land during their lifetime.

The brother, upon his death, devised his interest in the land to a friend. The sister later died intestate, and her sole heir inherited all of her property.

Who owns the undeveloped land? Quote the legal rule implicated…

Very tricky question!!! know the rules!

A

The sister’s sole heir owns a 2/3 interest and the brother’s friend owns a 1/3 interest in the land as tenants in common.

Due to the right of survivorship, a joint tenancy cannot be devised by will or inherited through intestate succession. However, a tenancy in common can be devised or inherited.

Here, the conveyance lacked unity of interest because the sister received a two-thirds interest and the brother received a one-third interest. This created a tenancy in common—not a joint tenancy. Therefore, the brother’s one-third interest passed to the friend (not the sister) upon his death and the sister’s two-thirds interest passed to her sole heir, as a tenancy in common.

How well did you know this?
1
Not at all
2
3
4
5
Perfectly
33
Q

A buyer entered into a contract to purchase a house from its owner. The contract called for the buyer to make equal monthly installment payments over 10 years. During that time, the owner was to retain title to the house and the buyer was granted the right to occupy the premises. Once the buyer made all of the required payments, the owner was to transfer ownership of the house to the buyer. The contract contained an acceleration clause under which all future installment payments were to become due in the event the buyer failed to timely make a required installment payment. Additionally, the contract included a forfeiture clause, which stated that time was of the essence and permitted the owner to terminate upon the buyer’s failure to timely make a required installment payment, regain possession of the house, and retain any payments already made by the buyer.

After making timely payments for seven years, the buyer failed to make three monthly payments. In accordance with his rights under the contract, the owner filed a summary ejectment action to evict the buyer from the house. The buyer appeared at the summary ejectment proceeding, and she tendered the missed payments.

The applicable jurisdiction treats an installment land contract as a mortgage, follows the lien theory of mortgages, and does not recognize a mortgagee’s right of strict foreclosure.

Should the court award possession of the house to the owner?

Read this one carefully! You didn’t read carefully last pass!

A

No, because there has not been a foreclosure sale.

In a jurisdiction that treats an installment land contract like a mortgage, a buyer in default may redeem the property by tendering to the owner the full balance due under the contract prior to foreclosure.

Notice that the buyer didn’t pay the full balance of the loan, just the past due amount.

How well did you know this?
1
Not at all
2
3
4
5
Perfectly
34
Q

A widower owned a house in fee simple absolute. His daughter is his only child. The daughter also has one child, the widower’s grandson. The grandson and his wife had just had their first child when the widower executed a will in which the house was devised to his daughter for her life and the remainder to his grandson’s children. The widower left the rest of his estate to a charity.

After the widower’s death, his grandson had a second child and the widower’s daughter died shortly thereafter. A year later, the grandson had a third child. The widower’s grandson recently died, survived by all three of his children.

The jurisdiction follows the common-law Rule Against Perpetuities as well as the Rule of Convenience.

Who now owns the house? Quote the legal rules implicated…

A

The first child and the second child.

The Rule of Convenience prevents the Rule Against Perpetuities from being applied to class gifts by closing class membership when any member of the class is entitled to immediate possession of a share in the class gift.

How well did you know this?
1
Not at all
2
3
4
5
Perfectly
35
Q

A man purchased undeveloped land with a bank loan secured by a mortgage on the property. The man recorded the deed, and the bank promptly recorded the mortgage. A year later, the man decided to sell the property to a wealthy widower. The widower purchased the property, recorded his interest, and assumed the mortgage. Several years later, the widower gave the property to his daughter. The widower did not tell his daughter about the mortgage but instead continued to make the mortgage payments. The deed, which contained no mention of the mortgage, was promptly recorded by the daughter.

When the widower died, he devised all of his real property to his daughter. He left the remainder of his estate to his son. Following the widower’s death, no one made payments on the bank loan, causing it to fall into default.

May the bank foreclose on the property? Quote the legal principles implicated…

A

Yes, because the bank recorded its mortgage.

Recording acts protect purchasers—not donees. The Shelter Rule gives donees who acquire property from a grantor protected by a recording act the same protection as the grantor under the recording act. Otherwise, the “first in time, first in right” rule applies.

How well did you know this?
1
Not at all
2
3
4
5
Perfectly
36
Q

A homeowner whose house sat on an irregularly shaped lot constructed a storage shed that by mistake rested entirely on his neighbor’s property. The storage shed stood there for 16 years before the neighbor discovered the mistake upon selling her property. The homeowner apologized and dismantled the shed. The neighbor then transferred title to her property to a buyer who promptly recorded the deed.

Six months later, the homeowner died, devising all of his real property interests to his son. When the son sought to construct a storage shed in the same location as the prior shed, the buyer objected and initiated a lawsuit against the son.

In the applicable jurisdiction, the statute of limitations for adverse possession is 15 years.

Which of the following is the buyer’s best argument that the land on which the original storage shed was located belongs to the buyer?

Read this one carefully! Take note to the call of the question!

A

The homeowner’s possession of this land was not hostile, assuming the jurisdiction follows the minority rule on hostility.

In most jurisdictions, possession is hostile if the adverse possessor objectively demonstrates an intent to claim the land. However, a minority of jurisdictions considers the possessor’s subjective intent.

Notice that the best argument must defeat the adverse possession elements which are present here.

How well did you know this?
1
Not at all
2
3
4
5
Perfectly
37
Q

What are common exceptions to enforceability to a due-on-sale clause affecting residential property?

A
  1. Devise, descent, or transfer to joint tenant upon death
  2. Transfer to spouse or child
  3. Transfer to ex-spouse in divorce
  4. Transfer to borrower’s living trust
  5. Creation of subordinate lien without occupancy rights
  6. Granting leasehold interest of less than 3 years without option to purchase
How well did you know this?
1
Not at all
2
3
4
5
Perfectly
38
Q

A homeowner bought a home with the proceeds of a loan from a thrift institution. The loan was secured by a mortgage on the home. Under the terms of the loan, the full amount of the outstanding loan obligation was to become due and payable if the home was sold or otherwise transferred without the prior permission of the thrift institution. The thrift institution recorded its mortgage.

Subsequently, the homeowner established a living trust and transferred ownership of her home to the living trust. The homeowner recorded this ownership transfer. Upon learning of the transfer, the thrift institution demanded that the homeowner pay the outstanding amount due on the loan immediately. When the homeowner refused, the thrift institution brought a foreclosure action to collect the full amount of the outstanding loan obligation.

Is the thrift institution likely to succeed?

A

No, because ownership of the home was transferred to the homeowner’s living trust.

A due-on-sale clause permits the mortgagee to demand payment in full of the remaining mortgage debt if the mortgaged property is transferred without the mortgagee’s consent. However, the transfer of residential property to the mortgagor’s living trust is exempt from this clause.

How well did you know this?
1
Not at all
2
3
4
5
Perfectly
39
Q

By statute, a jurisdiction provides: “Any judgment properly filed shall, for 10 years from filing, be a lien on the real property then owned or subsequently acquired by any person against whom the judgment is rendered.” In addition, the recording act of the jurisdiction reads, in its entirety, as follows: “No conveyance or mortgage of real property shall be good against subsequent purchasers for value and without notice unless the same be recorded according to law.” This act has been interpreted as not providing any grace period for recording a conveyance or mortgage.

An owner conveyed land by a warranty deed to his adult child. The child recorded the deed a week later. Three days after the conveyance to the child and without knowledge of it, a creditor of the owner properly filed a judgment against the owner. The creditor then filed suit against the owner and his child to foreclose on the judgment lien against the land.

If the court rules against the creditor, which of the following is the most likely reason?

Read the statutes carefully! Get a clear understanding of what they actually say.

A

The creditor is not a purchaser for value.

Judgment creditors are not purchasers for value since the attachment of a judgment lien to a debtor’s property is merely security for a preexisting debt—not payment of value.

How well did you know this?
1
Not at all
2
3
4
5
Perfectly
40
Q

A fitness company entered into a 10-year lease with the landlord of a gym facility. The lease required the fitness company to maintain the gym equipment in proper, working condition and to upgrade or replace any of the equipment as required by the safety guidelines for gymnasiums issued by a national organization of gymnasiums. In addition, the lease specified that all of the fitness company’s clients must sign a valid waiver releasing the current landlord from liability for any injury arising from their improper use of the gym equipment.

One year into the lease, the landlord transferred the remaining term of the fitness company’s lease to a large fitness conglomerate. The transfer occurred without the fitness company’s consent. The fitness company paid rent to the conglomerate, but the company stopped making its clients sign the liability waiver because the conglomerate did not require any of its gym members to sign one.

The conglomerate has brought an action against the fitness company to enforce this covenant in the lease.

Who will likely prevail?

A

The conglomerate, because the fitness company had required its clients to sign the waiver in the past.

A lease covenant can be enforced by an assignee-landlord if the covenant runs with the land—i.e., the original parties intended to bind their successors, the covenant touches and concerns the land, and there is privity of estate.

How well did you know this?
1
Not at all
2
3
4
5
Perfectly
41
Q

What is a right of first refusal? Explain.

A

A right of first refusal is a contractual right to purchase property before any other person if the owner later decides to sell, so it is a contingent future interest that is generally subject to the Rule Against Perpetuities (RAP).*

a right of first refusal is a contingent future interest

How well did you know this?
1
Not at all
2
3
4
5
Perfectly
42
Q

A grandmother had lived in her family’s mansion for her entire life, but she decided to sell the property and move into a smaller home. Desiring to keep the mansion in her family, the grandmother sold the mansion to her grandson at a below-market price. The grandmother included a right-of-first-refusal clause in the valid, written deed to her grandson, which the grandson signed. The clause stated that, in the event the grandson, his heirs, devisees, or assigns attempted to sell the property to a non-family member, the grandmother, her heirs, devisees, or assigns would have the opportunity to purchase the property before the transfer.

One year after the grandson purchased the property, he was approached by a buyer who offered him twice the price he had paid his grandmother. The grandson readily accepted and immediately sold the mansion to the buyer. The grandmother subsequently read about the sale in the local newspaper and brought an action against the buyer to enforce her right of first refusal.

The jurisdiction adheres to the common-law Rule Against Perpetuities.

Which of the following doctrines will help determine whether the grandmother will be able to enforce the right-of-first-refusal clause?

You didn’t know the rules implicated by the prompt! Get a clear understanding of the rules.

A

The Rule Against Perpetuities.

**Rights of first refusal **are generally subject to the Rule Against Perpetuities, so this contingent future interest is void if it there is any possibility that it could vest more than 21 years after some relevant life in being at the creation of the interest.

How well did you know this?
1
Not at all
2
3
4
5
Perfectly
43
Q

Discribe the effect of a mortage on a JT/ROS in a title theory state.

Is this the majority rule?

A

In a title theory jurisdiciton a mortgage constitutes a transfer of title, so granting a mortgage will sever a joint tenancy and the mortgagor-tenant becomes a tenant in common with the other joint tenant(s).

NO! The Lien Theory is of mortgages is the rule in majority of jurisdictions.

How well did you know this?
1
Not at all
2
3
4
5
Perfectly
44
Q

Discribe the effect of a mortage in a lien theory state

A

Lien theory (majority rule) – a mortgage is merely a lien on the property, so granting a mortgage does not sever the joint tenancy. However, severance will occur upon a foreclosure sale following a default.

How well did you know this?
1
Not at all
2
3
4
5
Perfectly
45
Q

The owner of a restaurant decided to pursue a different line of work, so he conveyed the restaurant to an up-and-coming chef. The owner executed a valid, written deed to the chef, who did not record the deed. The chef was talented, but he did not understand how to run a business, so his restaurant failed within a few months. A culinary school, in search of a new location to hold its cooking classes, purchased the restaurant from the chef. The chef executed a valid, written deed to the culinary school, and the culinary school promptly recorded the deed.

After the sale, but before the culinary school had a chance to occupy the restaurant space, the original owner noticed that the restaurant was vacant. The owner then sold the space to a fast-food chain, and the fast-food chain promptly recorded the deed. The owner did not tell the fast-food chain of his earlier conveyance of the restaurant to the chef, and the fast-food chain otherwise lacked actual knowledge of this conveyance.

Subsequently, the chef recorded the deed from the owner conveying the restaurant to him. When the fast-food chain attempted to take possession of the restaurant, it discovered that the culinary school had moved into the restaurant.

The fast-food chain has filed an appropriate action to quiet title against the owner, the chef, and the culinary school. The jurisdiction in which the restaurant is located applies a race-notice recording statute.

Who will likely prevail?

A

The fast-food chain.

A recorded deed that falls outside the chain of title is a “wild deed” that fails to give constructive notice to subsequent purchasers.

Here, the fast-food chain will prevail over the chef’s and the culinary school’s earlier interests if it is a BFP and the first to record. The fast-food chain purchased the restaurant with no actual knowledge of the prior interests and no inquiry notice because the restaurant was unoccupied. Therefore, the fast-food chain is a BFP if it also lacked constructive notice of the earlier interests.

How well did you know this?
1
Not at all
2
3
4
5
Perfectly
46
Q
A

A due-on-encumbrance clause gives the mortgagee the right to accelerate a mortgage obligation—i.e., to demand immediate payment of the full amount of the outstanding loan obligation, including interest—when the mortgagor obtains a second mortgage or otherwise encumbers the property.

How well did you know this?
1
Not at all
2
3
4
5
Perfectly
47
Q

A landowner died and left a piece of land to his three sons as joint tenants with the right of survivorship. The youngest son sold his interest in the property to the oldest son. The oldest son then died and left all of his real property interests to his daughter. The youngest son later died. Following the youngest son’s death, the middle son gave his interest in the property to a nephew.

The applicable jurisdiction continues to follow the common law with regard to joint tenancy.

Who owns the property?

sketch a quick diagram, this one is tricky!

A

The daughter and the nephew hold the property as tenants in common, with the daughter owning a one-third interest and the nephew a two-thirds interest.

A lifetime conveyance of a joint-tenancy interest destroys unity of title and severs the joint tenancy with respect to that interest. If there are more than two joint tenants, then severing one tenant’s interest does not affect the joint tenancy between the remaining tenants. And unlike a tenancy in common, a joint tenancy is not devisable.

How well did you know this?
1
Not at all
2
3
4
5
Perfectly
48
Q
A

Due to the doctrine of equitable conversion, a judgment obtained against a seller after the execution of a land-sale contract is not enforceable against the real property—even if the claim arose before the contract was executed.

How well did you know this?
1
Not at all
2
3
4
5
Perfectly
49
Q

A purse maker sought to market her line of “smart” purses that were compatible with a new handheld device. As part of her plans, the purse maker sought to purchase a brick-and-mortar store from which to sell her purses. The purse maker found a suitable store and entered into a contract with the owner of the store. The contract was in writing, signed by both parties, and stated the essential terms, including a closing date in 30 days. Due to the purse maker’s plan to sell her purses in advance of the release of the new handheld device, the contract stated that the closing date could not be delayed.

One week before the closing date, the purse maker discovered that the store was in violation of a zoning ordinance that mandated an updated version of the current fire sprinkler system. The owner promised that he would promptly update the fire sprinkler system and that, although it would not be finished by the closing date, it would be done in time for the grand opening of the store. In addition, the owner promised to provide a warranty deed upon closing to shield the purse maker from any potential liability stemming from the outdated fire sprinkler system. On the day of closing, the purse maker refused to close the land-sale deal.

In an action by the owner for specific performance against the purse maker, who will likely prevail?

A

The purse maker, because the owner did not provide marketable title to her on the closing date as stated in the land-sale contract.

A seller must deliver marketable title on the closing date when time is of the essence—i.e., (1) the contract states that “time is of the essence,” (2) circumstances indicate that the parties intended to strictly adhere to the closing date, or (3) one party notifies the other that time is of the essence within a reasonable time prior to closing.

How well did you know this?
1
Not at all
2
3
4
5
Perfectly
50
Q

A farmer informed his best friend, who had fallen on hard times, that he wanted to give the friend a small farm that the farmer had recently acquired. The friend told the farmer that he would be honored to own it. The farmer had his attorney prepare a proper deed transferring the farm to the friend. The farmer signed the deed, and his signature was notarized.

On his way to deliver the deed to the friend, the farmer stopped by his home for lunch. The farmer placed the deed on a small table in the entryway, along with his car keys and wallet. The farmer had lunch and then took a nap. While napping, the farmer died. The farmer’s adult son, from whom the farmer had been estranged for several years, claims that the deed is not valid. The friend contends that the farmer’s deed was effective to convey the farm to him.

The farm is located in a race-notice jurisdiction.

Did the farmer likely transfer his farm to the friend?

Be careful with this question! Look at the objective facts carefully!

A

Yes, because the farmer intended to deliver the deed to the friend.

A deed transfers ownership of real property when it is delivered by the grantor and accepted by the grantee. Delivery is shown by the grantor’s intent to make a present transfer of the property—which can be implied from the grantor’s words and conduct—and acceptance is presumed when the transfer is beneficial to the grantee.

How well did you know this?
1
Not at all
2
3
4
5
Perfectly
51
Q
A
How well did you know this?
1
Not at all
2
3
4
5
Perfectly
52
Q
A
How well did you know this?
1
Not at all
2
3
4
5
Perfectly
53
Q

Question ID# 9946

A buyer entered into a written contract to purchase real property from its owner. The buyer asked that the owner convey the property to the buyer and her brother as tenants in common. The owner noted that the buyer’s brother would need to attend the closing to sign the necessary paperwork. Because the brother lived in another state and could not attend the closing, the buyer brought her roommate to the closing instead. The roommate pretended to be the buyer’s brother and signed all the necessary paperwork with the brother’s name. The buyer paid the full purchase price, and the deed granting the buyer and her brother half interests as cotenants was recorded on the same day.

Unbeknownst to any of the parties, the evening before the closing, the buyer’s brother had died in a car accident. The brother’s valid probated will devised all of his property to his wife. The brother’s wife has brought an action against the buyer, who has taken sole possession of the property, and the original owner to quiet legal title to an undivided one-half interest in the property.

Who should the court find has legal title to the real property, and in what proportions?

A

One-half in the buyer and one-half in the original owner.

A deed that names a nonexistent grantee is void as to that nonexistent grantee. So if a nonexistent grantee was conveyed an interest in a tenancy in common, then the grantor would retain the nonexistent cotenant’s interest and have a tenancy in common with the other cotenant(s) named in the deed.

You chose: (Choice D) If a contracting buyer dies before the property is transferred, anyone entitled to the decedent’s property (e.g., an heir) can compel its transfer to him/herself. But here, the brother was not the buyer because he was never actually a party to the contract and never authorized the roommate to act as his agent. Consequently, the one-half interest deeded to the buyer remains with the original owner.

How well did you know this?
1
Not at all
2
3
4
5
Perfectly
54
Q

The owner of a commercial building obtained a nonrecourse, five-year loan from a lender and used the proceeds to fund another business project. The lender secured the loan with a mortgage on the building. Under the terms of the loan, monthly loan payments constituted only interest on the loan. The loan required a single payment of the principal amount (i.e., a “balloon” payment) at the end of the five-year period. Three years after the loan was made, the building was damaged by an unexpected hurricane. The owner did not repair the damage done by the hurricane, and he did not take action to protect the building from further damage. The contract was silent with regard to any obligation to repair the building.

At the end of the five-year period, despite an overall rise in property values for commercial buildings in the area, the value of the building was less than the amount of the balloon payment due to the building’s state of disrepair. The owner did not make the balloon payment. The lender sued to foreclose its mortgage. After the foreclosure sale, the lender filed an action against the owner personally.

If the court finds for the lender, which of the following is the most likely reason?

You don’t know the rules!

A

The owner committed waste.

Waste occurs when, for example, the mortgagor fails to maintain or repair the property in a reasonable manner.

Here, the hurricane damage does not constitute waste because it was due to natural forces, but the damage caused by the owner’s failure to prevent further damage could constitute waste.

A mortgagor in possession of the mortgaged property has a duty not to commit waste that would impair the mortgagee’s security interest in that property. If the mortgagor breaches this duty, the mortgagee can recover damages for the impairment.

How well did you know this?
1
Not at all
2
3
4
5
Perfectly
55
Q
A

If a seller cannot convey marketable title, the buyer can rescind the land-sale contract. But if the buyer accepts the land with the defect and the seller refuses to perform, then the buyer can (1) rescind the contract and seek restitution, (2) seek specific performance with an abatement of the purchase price, or (3) sue for damages.

How well did you know this?
1
Not at all
2
3
4
5
Perfectly
56
Q
A

Yes, but only to the extent of one-half of the payment made by the manager.

Under the doctrine of subrogation, a third party (subrogee) who pays another’s mortgage loan in full becomes the owner of the loan and the mortgage. The subrogee may therefore seek reimbursement from the debtor or enforce the mortgage.

How well did you know this?
1
Not at all
2
3
4
5
Perfectly
57
Q

A defendant, intending to frighten the plaintiff, went to the plaintiff’s place of work, a toy store. When the plaintiff was alone in the store, the defendant waved a knife at the plaintiff, saying, “I’m going to make sure you don’t molest any more children!” It turns out that the defendant had mistaken the plaintiff for someone else, who the defendant believed to have molested his nephew. The plaintiff, however, assumed that the knife was just one of the toys from a shelf in the store and that the defendant was just making a tasteless joke, so he laughed it off and told the defendant to get lost. Only later did the plaintiff realize that the knife had been real and that he had actually been in serious danger. As a result of looking back on the distressing event, the plaintiff has suffered from severe anxiety, leading to illness, which has caused him to miss work.

What tort doctrine is triggered by the question?

Why do the other intentional torts apply? Distinguish each!

A

Intentional infliction of emotional distress

A defendant is liable for intentional infliction of emotional distress (IIED) when the defendant, by extreme and outrageous conduct, intentionally or recklessly causes the plaintiff severe emotional distress. Conduct is considered extreme and outrageous if it exceeds the possible limits of human decency, so as to be entirely intolerable in a civilized society.

Assault requires that the defendant’s intentional conduct cause the plaintiff to anticipate an imminent, and harmful or offensive, contact. But here, the defendant did not cause the plaintiff to anticipate such contact because the plaintiff thought that the knife was a toy and that the defendant’s threat was a tasteless joke.

Battery is the intentional infliction of harmful or offensive contact. Here, the defendant merely waved the knife at the plaintiff, so no harmful or offensive contact occurred.

How well did you know this?
1
Not at all
2
3
4
5
Perfectly
58
Q

The owner of a bed-and-breakfast catered to wealthy city residents hoping to get some rest near the beach. His bed-and-breakfast had doors and windows that opened out directly onto the beach. On a busy holiday weekend, a guest at the bed-and-breakfast threw a party during which he fell out of a second-story window onto the sand below. His injuries were minor, but he filed suit against the owner, alleging that the owner was negligent in allowing the window to fully open on an upper-level floor.

During the proceedings, the guest provided evidence that within that same beach town, there were six bed-and-breakfasts, and all of them contained locks on their upper-level windows preventing them from opening more than four inches, although there was no statute requiring this precaution. The owner countered with evidence that no other guest had ever fallen out of a window, and that the windows had recently been inspected and given a positive safety rating. The guest then moved for judgment as a matter of law.

In a jurisdiction that has abolished the traditional rules of innkeeper liability, is the court likely to grant the guest’s motion?

You didn’t know the duty innkeepers owe to guests under the traditional or modern approach.

A

No, because the windows were recently inspected and noted to be safe.

Under the traditional common-law approach, innkeepers owed the highest duty of care to their guests and could be liable for even slight negligence. However, in most jurisdictions today, innkeepers only owe a duty to use ordinary care to protect their guests while they are on the premises.* Ordinary care is the care that a reasonably prudent person would use under the circumstances.

How well did you know this?
1
Not at all
2
3
4
5
Perfectly
59
Q

In tort law what is the mnemonic to help identify a special relationships where a modified duty of care may be expected?

A

Please Help Eliminate Safety Concerns Causing Injuries

Special relationships

imposing duty to protect others

Parent/child
Hospital/patient
Employer/employees
Shopkeeper/business invitees
Common carrier/passengers
Custodian/person in custody
Innkeeper/guests

How well did you know this?
1
Not at all
2
3
4
5
Perfectly
60
Q

The plaintiff, while driving his car, had been stopped at a red light at a busy intersection for approximately 15 seconds when he was struck from behind by a speeding taxicab. The plaintiff did not see the taxicab coming toward him in his mirrors because he had been looking down to send a text message. The jurisdiction where the accident took place, which is a contributory negligence jurisdiction, has enacted a statute prohibiting drivers from texting while driving. The statute was enacted after a study showed that most car accidents in the jurisdiction were caused by driving while texting. The plaintiff brings suit against the taxicab driver for damages for injuries resulting from the accident. The taxicab driver asserts the defense of contributory negligence, arguing that because the plaintiff was texting in violation of the statute, he was negligent per se.

Is the contributory negligence defense likely to be successful?

You picked an enticing wrong answer.

A

No, because the plaintiff’s own negligence was not the cause of the accident.

In traditional contributory negligence jurisdictions, a plaintiff’s failure to use reasonable care for his/her own safety is a complete defense to negligence. The defendant can establish the plaintiff’s negligence under the doctrine of negligence per se, but still must prove that the plaintiff’s negligence proximately caused the plaintiff’s harm.

Your first go at this question you picked: No, because the taxicab driver was not in the class of people the statute was designed to protect.

How well did you know this?
1
Not at all
2
3
4
5
Perfectly
61
Q

A motorcyclist traveling along a hilly road through the woods at dusk experienced sudden engine trouble. He stopped his motorcycle in the middle of the road and, as he was unable to get it started again, left it there and walked to find assistance. Two drivers, who were approaching the motorcycle from opposite sides of the road, failed to see the motorcycle because the motorcyclist had neglected to turn on his hazard lights and neither car’s automatic headlights had come on. As a result, both drivers simultaneously clipped the motorcycle, causing damage so significant that the motorcycle was totaled.

The motorcyclist subsequently filed a negligence action against the two drivers to recover damages for the loss of his motorcycle, which had been damaged beyond repair. At trial, it was established that both drivers had failed to act as reasonably prudent persons under the circumstances because their headlights were off. It was also established that the impact of either car alone would have been sufficient to total the motorcycle and that the motorcycle would have been visible to both drivers if its hazard lights had been turned on.

Is the motorcyclist likely to recover damages from the drivers?

A

Yes, because each driver’s conduct was a substantial factor in causing the motorcyclist’s damages.

Where multiple forces combined to cause the plaintiff’s harm and any one alone would have been sufficient to cause the harm, the test for actual causation is whether the defendant’s conduct was a substantial factor in causing the harm.

62
Q
A

Strict products liability claims can be brought against commercial suppliers or sellers—i.e., those in the business of manufacturing, selling, or otherwise distributing products of the type that harmed the plaintiff. However, service providers are not subject to strict products liability.

63
Q

A plaintiff purchased a new car from a car dealer. A computer chip in the engine of the car had been defectively manufactured, such that the plaintiff lost control of the car, causing the plaintiff to suffer injuries. Several entities were involved in the process through which the defective chip ended up in the plaintiff’s car. The defective computer chip was manufactured by a component company and then purchased by an engine builder. The engine builder then incorporated the chip into the engine and sold the engine to a car manufacturer. The car manufacturer then incorporated the engine into the car and sold the car to the car dealer. The plaintiff brought an action for strict products liability against the component company, the engine builder, the car manufacturer, and the car dealer.

From whom can the plaintiff recover for his injuries?

A

Any of the parties, regardless of whether the party was negligent.

Any commercial seller in the distribution chain—e.g., manufacturer, distributor, retailer—is subject to strict products liability if (1) the commercial seller’s product was defective when it left the commercial seller’s control and (2) that defect caused the plaintiff harm. Strict liability is imposed even if the commercial seller did not create or know about that defect.

64
Q

A chef who worked in a restaurant purchased an electric knife from a knife manufacturer that was specially designed to cut through meat. The instructions for the electric knife clearly stated that it should not be used on raw, hard vegetables, such as carrots or potatoes, because the moving blades on the electric knife would malfunction and could cause serious injury. One day, the chef’s friend asked to borrow the electric knife. The chef lent the electric knife to the friend and told him, “Don’t use the electric knife to cut raw vegetables.” Thinking that the advice was only to avoid dulling the blades, the friend disregarded the chef’s advice. When the friend used the electric knife to cut raw carrots, the moving blades on the knife malfunctioned, causing the friend to sustain a serious knife wound. The friend filed a negligence suit against the chef to recover damages for his injuries. The jurisdiction applies the common-law rules for contributory negligence and assumption of the risk.

Which of the following is the chef’s best defense?

A

The friend was contributorily negligent

Under the common-law rule for contributory negligence, the plaintiff’s failure to use reasonable care for his/her own safety is a complete defense to negligence—regardless of the percentage that the plaintiff’s own negligence contributed to the harm.

65
Q
A
66
Q

The defendant purchased a tiger in the hope of achieving his life’s ambition of performing in a Las Vegas show. The defendant kept the tiger locked in a cage specifically designed for such animals. Unbeknownst to the defendant, however, there was a manufacturing defect in the cage that caused the lock to spring open. The tiger escaped from the cage and ran onto the defendant’s yard, where the plaintiff, a painter hired by the defendant, was on a ladder painting the exterior of the defendant’s home. The plaintiff panicked when he saw the tiger, causing him to fall off the ladder. The tiger ran off the property without approaching the plaintiff. The plaintiff sustained significant injuries as the result of his fall. The plaintiff has sued the defendant. At trial, the defendant provided evidence that he exercised reasonable care in containing the tiger.

The jurisdiction recognizes the doctrine of contributory negligence.

Is the plaintiff likely to prevail at trial?

Be careful with this question. You were fooled by a relative easy question.

A

Yes, because the defendant is strictly liable for the plaintiff’s injuries.

The owner of a wild animal is strictly liable for harm that is caused by a plaintiff’s fearful reaction to the sight of an unrestrained wild animal or directly results from the wild animal’s abnormally dangerous characteristics.

67
Q
A

Under pure comparative negligence, a negligent plaintiff’s recovery is reduced by his/her proportionate share of fault. And if multiple defendants cause the plaintiff indivisible harm, several liability limits the plaintiff to recovering from each defendant the portion of damages that corresponds to his/her proportionate share of fault.

68
Q

The plaintiff brought his car to a mechanic for a tune-up. During the tune-up, the mechanic removed the fuse for the brake lights and inadvertently failed to replace it, causing the lights to stop working. As the plaintiff was driving his car home from the mechanic, he was involved in an accident. The plaintiff, seeing a friend walking along the road, slammed on his brakes to give the friend a ride. The defendant, who was driving the car behind the plaintiff, hit the plaintiff. The plaintiff sustained severe whiplash from the accident and has sued the defendant and the mechanic.

At trial, the jury determined that the plaintiff’s damages were $10,000 and that the defendant was 50% at fault, the mechanic was 25% at fault, and the plaintiff was 25% at fault. The jurisdiction recognizes pure several liability and partial comparative negligence.

How much can the plaintiff collect from the defendant?

A

$5,000

Under modified (or partial) comparative negligence, recovery is reduced by the plaintiff’s percentage of fault and barred if it exceeds 50%. If multiple defendants cause the plaintiff indivisible harm, then several liability limits the plaintiff to recovering the portion of damages that corresponds to each defendant’s share of fault.

69
Q

A professional football player was one of the most popular and well-known athletes on television. He regularly appeared in commercials endorsing sports equipment, cars, and performance-enhancing vitamins. After a recent drug test was administered to all professional football players, the football player’s test results were leaked to the public by an unknown source. The results showed that he tested positive for illegal performance-enhancing drugs. A local political activist, who campaigned against professional football players because she believed all of them used illegal drugs, stated in a televised interview that the leaked results showed that the football player was an “illicit drug user who will be suspended from playing professional football.”

A few days later, the drug-testing facility announced that it had mistakenly reported the football player’s test results and that he had not actually tested positive for the use of illegal drugs. By that time, however, the football player had already lost a number of lucrative advertising contracts.

If the football player sues the activist for defamation, who will prevail?

A

The activist, because the professional football player is a public figure.

A plaintiff who is public figure or official can recover for defamation only if the plaintiff proves that the defendant made a false statement about the plaintiff with actual malice—i.e., with knowledge or reckless disregard of the statement’s falsity.

70
Q
A

The warranty of marketable title is implied in all real estate contracts unless otherwise provided. This guarantees that, upon closing, the seller will convey the buyer title that is free from an unreasonable risk of litigation. Therefore, this is the standard that the court should apply.

71
Q

What are “Red flags” for marketable title issues?

A

Covenants
Easements
Leases
Liens
Gaps in chain of title
Boundary disputes
Existing zoning violations
Adverse possession

72
Q

The owner of a daycare business operated the daycare out of a rented building located next to a fenced-in playground. Under the daycare owner’s direction, the children at the daycare facility used the playground five days a week. The daycare owner did not own the parcel upon which the playground was located, nor did she know who did. Had she asked, the parcel owner would have given permission for the children to use the playground. The daycare owner padlocked the playground when not used by the daycare children in order to keep vagrants and other people out. After renting the building and using the playground for eight years, the daycare owner suddenly fell into a coma. The owner of the building that housed the daycare operation then rented the building to a childcare center, and the manager of the childcare center continued to use the playground just as the daycare owner had for the next four years.

Recently, the daycare owner died. Shortly thereafter, the owner of the playground parcel returned, introduced himself to the manager of the childcare center, and revealed his plans to build a high-rise condominium on the playground parcel. The manager has filed a suit seeking a declaration that she is the owner of the parcel.

The statutory period to obtain title by adverse possession in the jurisdiction is 10 years.

Which of the following is the best defense that the owner of the playground parcel can make against the lawsuit?

A

The childcare center’s manager has not possessed the playground for the statutory period.

The statutory period in this jurisdiction is 10 years, but the childcare center’s manager has used the playground for only 4 years. However, under the doctrine of tacking, an adverse possessor may tack on the predecessor’s time if there is privity between successive adverse possessors. Privity is satisfied if the possessor takes by nonhostile means (e.g., by descent, devise, contract, deed).

73
Q

The owner of a warehouse entered into a very basic contract to transfer title and possession of the warehouse to a buyer. The contract did not mandate the type of deed that the owner was required to convey to the buyer at closing and did not require the owner to transfer his title to the warehouse free of all defects. Additionally, the contract did not require either party to insure the building against casualty losses.

After the contract was executed, the owner allowed his casualty insurance on the warehouse to lapse even though he retained possession of the warehouse during the executory period. Several days prior to closing, the warehouse was destroyed by a tornado. As a consequence, the buyer refused to pay the purchase price specified in the contract to the owner. The owner then filed suit against the buyer to compel the buyer to honor her contractual obligation to pay the purchase price to the owner.

Which of the following is likely to be the most important issue in awarding judgment to a party?

A

Whether the applicable jurisdiction has adopted the Uniform Vendor and Purchaser Risk Act.

A minority of jurisdictions have adopted the Uniform Vendor and Purchaser Risk Act. Under this act, the risk of loss remains with the seller until the buyer takes possession of or receives legal title to the property. Since the owner (seller) had possession of the warehouse at the time it was destroyed, the most important issue in awarding judgment is whether the jurisdiction has adopted the Uniform Vendor and Purchaser Risk Act.

Be sure to review the “Uniform Vendor and Purchaser Risk Act”. You were not familiar with it last time.

74
Q

The owner of an undeveloped lot needed money. The fair market value of the lot was $150,000. The owner approached an investor about borrowing $75,000. The investor agreed on the condition that the owner convey the lot to the investor outright by warranty deed. In exchange, the investor would lend the owner the $75,000, allow the owner to remain in possession of the lot, and orally agree to reconvey the lot to the owner once the loan was paid in full. The owner reluctantly agreed and conveyed the lot to the investor as agreed. The investor then paid the owner $75,000 and promptly and properly recorded the deed. A year later, the owner defaulted on the loan with a remaining balance of $50,000.

Which of the following best states the parties’ rights in the lot? What kind of interest is created?

A

The investor may foreclose on the lot because the transaction is treated as an equitable mortgage.

An equitable mortgage can be established when a debtor gives an absolute deed—i.e., a deed that is free of encumbrances and transfers unrestricted title to property—to a lender with the intent to secure a loan. The debtor-grantor must prove by clear and convincing evidence that the deed was intended as security for a loan—not as an outright transfer. The deed recipient, like any other lender, may then bring a foreclosure action if the debtor defaults.

last time you chose: The investor may evict the owner from the property following the owner’s default.

This choice is wrong because the transaction created an equitable mortgage—not an outright transfer of the lot. Therefore, the investor cannot immediately evict the owner. The lender must first follow any applicable foreclosure procedures.

75
Q

A landowner owned a three-acre plot of land in a remote location. His plot abutted a two-acre wooded property that contained an artesian spring owned by an out-of-state investor who had never developed or visited the land. Hoping to make a substantial profit by making his property sound more appealing, the landowner altered the existing deed for his three-acre plot so that the deed indicated that his parcel was five acres in size and included the wooded property with the artesian spring.

A buyer offered to pay $50,000 for the five-acre parcel and expressed that $10,000 per acre was quite reasonable. The landowner accepted and signed the altered warranty deed, conveying the five acres to the buyer. The landowner mailed the deed to the buyer, and the buyer subsequently took possession of the land. However, the buyer did not record the deed. A year later, the buyer learned that the out-of-state investor held title to the two acres of property that contained the artesian spring.

Is the buyer entitled to ownership of the five-acre parcel?

A

No, because the landowner altered the deed and that deed is therefore void.

A void deed is invalid at its inception and conveys no title to the grantee. As a result, a void deed is unenforceable even if it is relied upon by a bona fide purchaser—i.e., one who purchases a property interest without notice of another’s prior interest in the property.

76
Q

What are some exceptions to the fair housing act?

A

Act does not apply to:

  • religious organizations
  • private clubs that incidentally provide lodging to members
  • familial status for senior housing
  • owner of ≤ 3 single-family dwellings*
  • owner-occupied dwellings with ≤ 4 units*
  • sale/rental of single-family home by private owner
77
Q

When does the fair housing act apply housing act apply

A

The Fair Housing Act prohibits discriminatory actions in sale/renting of dwelling based on race, color, religion, sex, handicap, national origin, or familial status—including:

  • refusing bona fide offer
  • discriminating in terms, conditions, privileges, or services of purchase/rental
  • advertising preference for or discrimination against buyer/renter
  • falsely representing dwelling’s availability for inspection, sale, or rental
  • inducing sale/renting with discriminatory representations about neighborhood
  • denying reasonable modifications to accommodate handicap at occupant’s expense
78
Q

To pay for a home in the mountains, a retiree executed a mortgage with the seller of the home. Due to a clerical error, the seller’s mortgage was never recorded. Years later, the retiree executed a second mortgage on the home with a bank to make improvements, and that mortgage was immediately recorded. As part of the mortgage-approval process, the retiree had to disclose his first mortgage on the property. Shortly thereafter, the seller discovered that his mortgage had not been recorded and recorded it.

A year later, the retiree executed a third mortgage on the home after he realized the full effect his retirement was having on his finances. The retiree then went nearly a year without making mortgage payments, so a foreclosure action was initiated.

In a race-notice jurisdiction, which of the following is a correct statement about the first mortgage?

Watch out! This is a tricky one. Make sure you know the rules!

A

A PMM is a mortgage granted to the seller of real property if the mortgage is given as part of the same transaction in which title is acquired (as seen with the seller’s mortgage here).* A PMM has priority over liens that arose prior to the PMM regardless of whether the PMM was recorded. But a PMM does not necessarily have priority over subsequent liens. Instead, the recording act (or, if there is no recording act, the “first in time” rule) will control.

In this race-notice jurisdiction, a subsequent purchaser has priority over a prior conflicting interest if the purchaser (1) took the interest without notice of the prior interest and (2) recorded first. The bank’s mortgage (second) was recorded first. But since the seller’s mortgage (first) was disclosed during the bank’s mortgage-approval process, the bank had actual notice of that mortgage. Therefore, the first mortgage has priority over the second mortgage (Choices B & C).

In this race-notice jurisdiction, a subsequent purchaser has priority over a prior conflicting interest if the purchaser (1) took the interest without notice of the prior interest and (2) recorded first. The bank’s mortgage (second) was recorded first. But since the seller’s mortgage (first) was disclosed during the bank’s mortgage-approval process, the bank had actual notice of that mortgage. Therefore, the first mortgage has priority over the second mortgage (Choices B & C).

79
Q

A tenant leased a set of 10 commercial storefronts spanning two city blocks. The lease was for a term of five years and complied with all relevant statutes. The lease was silent as to the effect of condemnation by the city.

Three years into the lease, the city properly took one of the city blocks for public use pursuant to eminent domain and compensated the landlord accordingly. The city demolished five storefronts and began developing a public park. Upon this condemnation, the tenant stopped paying rent for all 10 storefronts. In an appropriate action, the landlord sued the tenant for the unpaid rent on all 10 storefronts.

Is the landlord likely to succeed?

You didn’t know the rules implicated here! Brush up on the relevant rules

A

Yes, but only for half the amount, because the tenant is entitled to compensation.

Upon a partial condemnation, a tenant must continue to pay rent but is entitled to compensation for the portion of the leased property that was taken. In contrast, upon a complete condemnation, the tenant is discharged from his/her rent obligation and is entitled to compensation for the taking.

80
Q

A landlord owned a luxurious property located in the hills that he typically rented out to famous celebrities and athletes. The property consisted of a 10-bedroom mansion, a pool, and tennis courts. The tennis courts were built into the side of the hill and supported on one side by a number of cement pillars.

The landlord rented the property to a tennis star under a one-year lease. She was training for a big tennis tournament scheduled toward the end of the lease period. One month into the lease, the local government condemned the tennis courts because the pillars supporting the tennis courts had not been retrofitted for earthquake safety. The landlord did not know that the pillars failed to meet earthquake-safety standards.

There was no provision in the lease regarding condemnation, and the only provision regarding the tennis courts pertained to the tenant’s obligation for basic upkeep and maintenance of the tennis courts. The retrofitting process would take approximately one year to complete. The tennis star immediately moved off the premises and refused to pay any rent.

If the landlord brings an action against the tennis star to recover the rent, who will likely prevail? Quote the relevant rule.

You got this one correct on the first pass but stay sharp on the rule[s] implicated.

A

A partial eviction occurs when a tenant is prevented from possessing or using a portion of the leased premises. The tenant is excused from paying rent for the entire premises if the landlord was responsible for the partial eviction. But here, the landlord is not responsible for the condemnation, so the tennis star must still pay rent.

81
Q

The owner of a building died. By the terms of his will, the owner devised the building to a religious organization “for so long as it is used for religious purposes.” The owner then devised all of his remaining real property interests to his companion. By law, the owner’s son was his only heir.

Ten years later, the religious organization ceased to use the building for religious purposes. The religious organization executed a quitclaim deed of the building to a developer in exchange for valuable consideration. The developer plans to convert the building to commercial uses. The developer has brought an action to quiet title to the building against the companion, the son, and the religious organization.

For whom should the court rule?

You got this question right, but for the wrong reason. Brush up on the interest created by the language in the conveyance.

A

The companion.

Here, the owner’s will conveyed an FSD in the building to the religious organization (to the religious organization “for so long as it is used for religious purposes”), and he did not pass the accompanying future interest to a third party. As a result, the owner retained a possibility of reverter, which was freely alienable by the owner during life and upon death. The owner devised his possibility of reverter, along with his remaining real property interests, to the companion by will—leaving no real property interest for the son to inherit.

82
Q

On April 1, a buyer and a seller executed a valid land-sales contract for a warehouse, with closing set for May 1. The buyer paid a deposit equal to 20% of the sales price at the signing. On April 15, the buyer discovered that the seller did not own the warehouse. The seller assured the buyer that it had a contract to purchase the warehouse from a third party and would possess title to the warehouse by the May 1 closing date. The seller had scheduled a closing with the third party to take title to the warehouse on April 30, but the third party did not appear at the closing. The seller notified the buyer that it could not timely secure title to the warehouse and would need to reschedule the May 1 closing date, offering an alternative closing date of May 3. The buyer refused to accept the rescheduled date.

On May 1, after the seller failed to appear for the closing, the buyer filed suit against the seller for rescission of the land-sales contract. On May 2, the seller acquired title to the warehouse and filed a counterclaim against the buyer for specific performance of the land-sales contract.

How should the court rule?

A

Here, the seller did not have title to the warehouse when it entered the contract or by the May 1 closing date (Choices C & D). However, there is no indication that time was of the essence. And since the seller did acquire marketable title to the warehouse on May 2 and was ready to close on May 3—a reasonable time after closing—the buyer cannot rescind the contract. The court should therefore order specific performance of the land-sales contract.On April 1, a buyer and a seller executed a valid land-sales contract for a warehouse, with closing set for May 1. The buyer paid a deposit equal to 20% of the sales price at the signing. On April 15, the buyer discovered that the seller did not own the warehouse. The seller assured the buyer that it had a contract to purchase the warehouse from a third party and would possess title to the warehouse by the May 1 closing date. The seller had scheduled a closing with the third party to take title to the warehouse on April 30, but the third party did not appear at the closing. The seller notified the buyer that it could not timely secure title to the warehouse and would need to reschedule the May 1 closing date, offering an alternative closing date of May 3. The buyer refused to accept the rescheduled date.

On May 1, after the seller failed to appear for the closing, the buyer filed suit against the seller for rescission of the land-sales contract. On May 2, the seller acquired title to the warehouse and filed a counterclaim against the buyer for specific performance of the land-sales contract.

How should the court rule?

83
Q

One month ago, a buyer signed a contract with a seller to purchase real property for $100,000. Upon signing the contract, the buyer paid the seller $5,000 as a deposit toward the purchase price. The contract contained a clause providing that the seller was permitted to retain the deposit as liquidated damages if the buyer defaulted on the agreement. The contract specified the date for closing, but on that date, the buyer informed the seller that he had been unable to attain financing for the purchase and that he would not be able to purchase the property. The buyer demanded the return of his deposit, but the seller refused. Instead, the seller sold the property to another buyer for $96,000.

The buyer has brought an action against the seller to recover his deposit.

For whom should the court rule?

You chose the right answer off instinct. Let’s see if you can answer correctly again… This time explain your answer.

A

The seller, because the buyer breached the contract.

Real-estate contracts usually require the buyer to make a deposit of a portion of the purchase price (i.e., an “earnest money” deposit). A liquidated damages clause is also often included, which allows the seller to retain the buyer’s deposit if the buyer breaches the contract and refuses to purchase the property. This clause is generally enforceable when the amount of liquidated damages is reasonable—e.g., no more than 10 percent of the purchase price. But when evaluating reasonableness, courts may also consider:

  • the sophistication of the buyer
  • the nature of the transaction (commercial v. residential) and
  • whether the seller suffered an actual loss (if not, courts may refuse to enforce the clause).
84
Q

What are remedies for a seller of real estate when a buyer breaches?

A
85
Q

What are remedies for a seller of real estate when a buyer breaches?

A
86
Q

A homeowner purchased a residence from its original owner, who agreed to finance the sale in exchange for a mortgage on the residence. The homeowner later obtained a loan from a bank to add a family room to his residence. In addition to the homeowner’s note promising to repay the loan, the bank demanded and the homeowner granted a mortgage on the property. Subsequently, the homeowner obtained a home equity loan from a credit union to remodel the kitchen. The loan was secured by a mortgage on the residence. All mortgages were promptly and properly recorded.

The homeowner, while continuing to make timely payments with respect to the loans from the bank and the credit union, failed to make the required payments to the original owner of the residence. The original owner filed a foreclosure action with respect to her mortgage. The credit union was made a defendant to this action, but due to an oversight, the bank was not made a defendant. At the judicial foreclosure sale, the property was sold for its fair market value. Proceeds from the judicial sale equaled the homeowner’s obligation to the original owner.

The jurisdiction permits a mortgagee to obtain a deficiency judgment.

What is the bank’s right with respect to its mortgage on the residence?

A

The bank’s mortgage continues, because the bank was not made a party to the foreclosure action.

Lien priority is generally determined by the “first in time, first in right” rule. Therefore, lien priority here is as follows: the original owner’s, the bank’s, then the credit union’s. As the foreclosing mortgagee, the original owner therefore had to give notice to the bank and the credit union (the junior-interest holders) and make them parties to her foreclosure action. The credit union was notified and joined, but the bank was not. Therefore, the bank’s mortgage continues after the foreclosure sale.

87
Q

February 2017 MPT-2 File: In re Guardianship of Henry King

Try this one Again! Organization is the Key. You can do It! think like a lawyer!

A

Try this MPT again!

88
Q

A plaintiff was injured when a roller coaster she was riding at an amusement park ran off the rails and crashed. After conducting a thorough investigation of the accident, the plaintiff was unable to determine what had caused the accident, but she did learn that the amusement park had failed to register with the county tax commission as required by state law.

Which of the following would be the strongest basis for the plaintiff’s tort claim?

A

Res ipsa loquitur.

Res ipsa loquitur permits an inference of negligence when the plaintiff’s harm was the type usually caused by negligence and evidence tends to eliminate other potential causes of that harm.

Last time you chose strict liability

Strict liability for a defective product is only imposed on commercial sellers (e.g., manufacturers, distributors, retailers)—not amusement park ride operators.

89
Q

A mother and father who recently divorced had one child. As part of their custody agreement, the judge awarded primary custody of the child to the mother and granted weekend visitations to the father. Soon after the divorce, the mother remarried another man, who treated the child like his own. The father became extremely angry when he learned about the mother’s remarriage, so he decided to cause her emotional distress. The father appeared at the mother’s house when he was supposed to return the child from a weekend visitation. When the mother and the stepfather answered the door, the father told them that the child had been in a car accident and was in a coma. In fact, the child was neither in an accident nor in a coma.

Both the mother and the stepfather suffered severe emotional distress as a result of the father’s news. Each of them sued the father for intentional infliction of emotional distress. The father has moved to dismiss the stepfather’s claim, arguing that his conduct was directed at the mother alone.

Should the judge grant the father’s motion to dismiss? What fact[s] do the question turn on?

A

No, because the stepfather is a close relative to the mother.

defendant whose extreme and outrageous conduct has harmed a third party may be liable for intentional infliction of emotional distress if (1) the plaintiff contemporaneously perceived that conduct, (2) the plaintiff was closely related to the third party, and (3) the defendant knew of the plaintiff’s presence and that relationship.

last time you said no, because the target of the father’s conduct is irrelevant for such a claim.

The target of the father’s conduct is relevant to the stepfather’s claim since generally only the actual target of the defendant’s extreme and outrageous conduct can recover damages for IIED. However, the stepfather contemporaneously perceived the conduct, he is close to the mother and child, and the defendant knew he was present.

90
Q

In a mall parking lot, two drivers simultaneously backed up and struck the other’s car. Neither driver suffered physical injuries, but the plaintiff’s luxury sedan sustained $10,000 in damage and the defendant’s oversized pickup truck sustained $1,000 in damage. The plaintiff filed a negligence action against the defendant to recover for the damage to her sedan. In his answer, the defendant filed a counterclaim to recover for the damage to his truck.

At trial, the jury determined that the plaintiff was 70% at fault and the defendant 30% at fault for the accident and that both parties are entitled to recover damages. The applicable jurisdiction has enacted a pure comparative-fault statute.

What is the total amount of damages that the plaintiff can recover?

A

$2,300

In a pure comparative-fault jurisdiction, when the plaintiff and the defendant are both entitled to recover damages, the plaintiff’s recovery is reduced (i.e., offset) by the defendant’s recovery (and vice versa).

Here, the jury determined that the plaintiff was 70% at fault and the defendant 30% at fault for the accident. Since this is a pure comparative-fault jurisdiction, the plaintiff’s recovery will be reduced by her proportionate share of fault ($10,000 - $7,000 = $3,000) (Choice D). And the defendant’s recovery will be reduced by his proportionate share of fault ($1,000 - $300 = $700). Since the jury determined that both parties are entitled to recovery, the plaintiff’s recovery will be reduced by the defendant’s recovery ($3,000 - $700), for a total recovery of $2,300.

You didn’t have a clear understanding of the rules last time you tried this question. Remember in a pure comparative fault jurisdiction, both

91
Q

A man and a woman lived directly across the street from each other. Early one morning before sunrise, the man and the woman backed their cars out of their driveways too quickly and simultaneously struck the other person’s car. Neither suffered physical injuries, but the woman’s car sustained $10,000 in damage and the man’s car sustained $1,000 in damage.

The woman filed a negligence action against the man to recover for the damage to her car. In his answer, the man filed a counterclaim to recover for the damage to his car. At trial, the jury determined that the woman was 55% at fault and the man 45% at fault for the accident and that both parties successfully established their claims. The jurisdiction follows a rule of modified comparative negligence.

What is the total amount of damages that the woman can recover? Explain?

A

Nothing!

Almost all jurisdictions have adopted some form of comparative negligence (i.e., comparative fault). There are two forms of comparative fault:

Pure comparative negligence (default rule on the MBE) – the plaintiff’s recovery is reduced by his/her percentage of fault

Modified comparative negligence (followed in this jurisdiction) – the same as pure comparative fault, except that the plaintiff’s recovery is barred if his/her fault exceeds 50%*

Remember, in a modified comparative-negligence jurisdiction the plaintiff or the defendant is barred from recovery if his/her percentage of fault exceeds 50%.

92
Q

A politician suffered from chronic social anxiety that caused him to have debilitating panic attacks before public speaking events. During an election year, the politician embarked on an extensive campaign tour across the country and asked his doctor about possible medications to alleviate his social anxiety. The doctor prescribed a popular prescription drug called CalmX that had helped many of her patients overcome their anxiety disorders.

The manufacturer of CalmX had warned all prescribing doctors about its side effect, which caused “extreme flatulence” when patients consumed a rare imported cheese while using the drug. The manufacturer’s instructions that accompanied the drug did not warn consumers about this side effect because it was extremely rare.

The politician successfully took CalmX during the campaign tour. However, on the day of a critical televised debate, he consumed a large amount of the rare imported cheese. As a result, the politician experienced extreme flatulence during the debate. The video of the debate went viral and caused him to lose the election.

The politician has brought a strict products liability action against the manufacturer of CalmX.

Is the politician likely to prevail?

A

No, because the manufacturer informed prescribing doctors about CalmX’s side effect.

Under the learned-intermediary rule, a manufacturer of a prescription drug or medical device will not be held strictly liable for inadequate warnings or instructions if the manufacturer warned the prescribing physician about the risk of harm associated with that product.

93
Q
A

To successfully assert the merchant’s privilege as a defense to false imprisonment, a defendant-merchant must have detained the plaintiff on or in the immediate vicinity of the merchant’s premises.

94
Q

A man and his friend decided to drive downstate to watch the homecoming football game at their alma mater. The man drove the friend to the game and refused any compensation from her. On the way home, the man failed to notice a stoplight and ran it, which caused a collision with another vehicle. The friend was severely injured as a result of the collision and sued the man for her injuries.

The jurisdiction where the friend’s suit was filed recently enacted a guest statute with regard to a driver’s duty to persons riding in the driver’s vehicle.

Will the friend be likely to recover damages for her injuries? What is a guest statute? Explain!

A

No, because the man was merely negligent.

In most jurisdictions, automobile drivers owe a duty of ordinary care to guests (who ride free) and passengers (who pay money for the ride). But a minority of jurisdictions have enacted “guest statutes,” under which an automobile driver’s only duty to guests is to refrain from gross or wanton and willful misconduct. Guest statutes do not absolve automobile drivers from liability for injuries to their guests. Instead, such statutes limit a driver’s liability by requiring that the driver have acted recklessly before a guest can recover damages.

Be careful with this question. Last time you didn’t know what a guest statute was. You chose: Yes, because the man owed his friend a duty to exercise ordinary care. Make sure your know this concept.

95
Q
A

The attractive nuisance doctrine specifically protects trespassing children, so this will not cause the child’s suit to fail.

96
Q

As a safety measure, a city enacted an ordinance imposing a fine for trespassing on construction and roadwork sites marked with orange caution tape. The city’s construction team marked a city block with orange caution tape to divert all cars and pedestrians while the team repaired potholes. A pedestrian wanted to walk down the marked-off street. The street was full of construction equipment, but because it appeared that construction had not yet started, the pedestrian stepped over the orange caution tape and walked down the street. Distracted by the equipment around him, the pedestrian stepped into an open manhole and landed on a construction worker who was inspecting the sewer under the construction site. Both parties were injured in the accident.

The construction worker has sued the pedestrian for negligence to recover damages for his injuries in a jurisdiction that applies the minority approach to negligence per se.

What is the legal effect of the pedestrian’s walking on a street marked with caution tape?

A

It creates a rebuttable presumption that the pedestrian breached a duty.

Under the doctrine of negligence per se, the majority approach is that duty and breach can be conclusively. However, under the minority approach, the violation of a statute or ordinance is merely evidence of negligence that creates a rebuttable presumption that the defendant breached a duty of care.

You didn’t know the minority approach and got this question wrong. Make sure you know the rules.

97
Q

Strict products liability

A

A commercial seller is subject to strict products liability when (1) a defective product harms a foreseeable plaintiff when it was used in an intended or reasonably foreseeable way and (2) the defect existed at the time the product left the commercial seller’s control.

98
Q

The owner of an abandoned building knew that many persons experiencing homelessness stayed in the building for shelter. Because he sympathized with their plight, the owner decided not to have the homeless removed from the building and left it in its current state because he had no plans to remodel it. Despite knowing that the building had dilapidated, rotting, and uneven floors, the owner never posted signs or informed those who stayed in the building about its uneven floors.

A man experiencing homelessness who tripped on an uneven floor and sprained his ankle brought suit against the owner to recover damages for his injury. In his defense, the owner argues that he had no obligation to take action to provide a safe environment to the man.

The jurisdiction has abolished traditional rules of landowner liability.

Will the owner’s defense likely be successful?

A

No, because the owner had a duty to protect the man from unsafe conditions.

Traditionally, the duty owed to land entrants depended on their status on the land (e.g., trespasser). But under the modern approach, land possessors owe all land entrants—except flagrant trespassers—a duty of reasonable care to protect them from foreseeable risk of harm. A flagrant trespasser is one who enters another’s land without permission and whose entry is particularly egregious—e.g., entry that results in commission of a crime.

99
Q

Immediately before leaving a party, a college student ate a cookie that, unknown to her, was laced with a hallucinogen. On the drive home, she lost control of her car as a consequence of the hallucinogen and struck another vehicle. The driver of the other vehicle sued the student under a negligence theory to recover for damages caused by the accident.

At trial, the judge instructed the jury that, under the reasonable-person standard, the student’s driving was to be judged in light of the care that a sober person in her situation would have exercised.

Has the judge correctly instructed the jury?

A

No, because the student unknowingly ingested the hallucinogen.

In negligence cases, a voluntarily intoxicated person is held to the same standard as a reasonably prudent sober person. In contrast, the conduct of an involuntarily intoxicated person will be measured by the standard of a reasonably careful person with the same level of intoxication.

Whether the judge correctly instructed the jury turns on whether the student was voluntarily or involuntarily intoxicated. Concluding that the student was merely intoxicated is insufficient to make this determination.

100
Q

The owner of a restaurant was responsible for printing new dinner menus every day after speaking with the restaurant’s chef in the morning. The menu always indicated in a footnote that all dishes were prepared in a kitchen that also worked with nuts and animal products. One morning, after speaking with the chef, the owner created a dinner menu that labeled a chicken dish as “soy, nut, and dairy free.” However, the chicken dish contained pecan crumbs.

That evening during dinner service, a patron of the restaurant told his waiter that he was allergic to nuts. Based on the menu, the waiter suggested that he order the chicken dish. The patron noted the warning in the footnote on the menu, but knowing that his own allergy was not severe enough to react to mere trace amounts of nuts, he ordered the chicken dish. After eating a few bites of the dish, he suffered a severe allergic reaction and was rushed to the hospital.

The patron filed a negligence suit against the owner of the restaurant to recover damages for his injuries. The above facts are undisputed at trial, and at the close of the patron’s case, the owner filed a motion for a directed verdict.

The jurisdiction has abolished traditional rules of landowner liability.

If no other evidence has been introduced, should the owner’s motion be granted?

A

No, because a jury could conclude that the owner breached her duty of reasonable care to the patron.

For a negligence claim, the plaintiff must prove that the defendant (1) owed the plaintiff a duty that (2) the defendant breached, which (3) caused the plaintiff harm that (4) entitles the plaintiff to damages. Under the modern approach, land possessors owe a duty of care to all land entrants (except flagrant trespassers).

You didn’t know the rules implicated by the facts!

Assumption of the risk is a defense to negligence if the plaintiff voluntarily accepted a known risk of harm. Here, the patron voluntarily accepted the risk that all dishes were prepared in a kitchen that used nut products. However, he did not voluntarily assume the risk that the chicken dish contained nuts since it was labeled “nut … free” on the menu.

A court should grant a defendant’s motion for a directed verdict if the plaintiff fails to present legally sufficient evidence to support every element of the claim—i.e., no reasonable jury could find in the plaintiff’s favor

101
Q
A

Zone of danger is a theory of recovery for negligent infliction of emotional distress—not IIED.

102
Q
A

A defendant whose extreme and outrageous conduct has harmed a third party may be liable for intentional infliction of emotional distress if (1) the plaintiff contemporaneously perceived that conduct, (2) the plaintiff was closely related to the third party, and (3) the defendant knew of the plaintiff’s presence and that relationship.

103
Q

A plaintiff is a candidate for state treasurer. The defendant has a long-standing grudge against the plaintiff. In an effort to dig up dirt on the plaintiff, the defendant hacked into the plaintiff’s personal email account and discovered emails that contained lewd pictures of the plaintiff engaging in sexual conduct with a former staff member, taken while the plaintiff was married. The defendant called the plaintiff on the phone and said to the plaintiff, “I hate you. I have always hated you. I just emailed your wife and your two top advisors, telling them that you are an adulterer. And tomorrow, I am going to drive over to your house and punch you right in the nose.” The plaintiff later confirmed that the defendant had emailed his wife and two top advisors to inform them that the plaintiff had committed adultery.

Under which of the following tort doctrines will the plaintiff be likely to recover?

A

Intrusion upon seclusion

ntrusion upon seclusion is an invasion of privacy that occurs when a defendant intentionally intrudes on a plaintiff’s private affairs in a manner that would be highly offensive to a reasonable person—e.g., hacking into an email account. As a result, the plaintiff can likely recover damages from the defendant for intrusion upon seclusion.

You didn’t now the rules for this tort! You chose: Public disclosure of private facts. A privacy claim for the public disclosure of private facts requires that the private matter be publicized—i.e., communicated to the public at large or to so many people that it is substantially certain to become public knowledge. Here, the plaintiff’s affair was not substantially certain to become public knowledge because the defendant only emailed the plaintiff’s wife and two closest advisors about it.

104
Q
A
105
Q

A skier was skiing down a mountain at a ski resort. As the skier turned a corner, she noticed an injured snowboarder lying unconscious and half-covered with snow at the edge of a difficult trail. Due to her reasonable speed, the skier was able to approach the snowboarder and stop without hitting him. She saw that he appeared to have suffered a compound fracture of his leg. The skier also recognized that the snowboarder was in grave peril, but because she had no cell phone and did not know how to administer first aid, she simply continued to ski down the mountain.

Less than an hour later, another skier came down the trail. Because he was skiing at an unsafe speed, he was unable to stop and struck the snowboarder. The injured snowboarder suffered a concussion from the crash. It was later established that the snowboarder had broken his own leg when he recklessly tried to do a flip on the difficult trail.

If the snowboarder sues the first skier to recover damages for his concussion, what is the first skier’s best defense?

A

The first skier took no action to place the snowboarder in greater peril than when she found him.

A defendant generally has no duty to aid a plaintiff who is at risk of physical harm unless the defendant’s conduct created that risk. When this occurs, the defendant has a duty to use reasonable care to prevent further harm to the plaintiff.

Last time you chose: The second skier’s negligence caused the snowboarder’s concussion. Although the second skier’s negligence caused the snowboarder’s concussion, this is not the first skier’s best defense because it merely helps to establish a claim against the second skier—not rebut the claim against the first skier.

106
Q

Negligence rules of recovery visual

A
107
Q

A mechanic at an auto shop accidentally spilled some gasoline in the shop’s parking lot and failed to clean it up. A customer walking across the parking lot ignited the gasoline when he accidentally dropped his cigarette lighter onto the ground while trying to light a cigarette. A nearby pedestrian suffered severe burns to her legs as a result of the fire caused by the ignited gasoline.

Can the mechanic be held liable for negligence for the harm to the pedestrian?

You got this one right but it was a lucky guess. Review the rules implicated

A

The chain of proximate causation can be broken by a superseding cause—i.e., an unforeseeable, intervening act or force that occurred after the defendant’s negligence and contributed to the plaintiff’s harm. But negligent intervening acts—e.g., accidentally dropping a cigarette lighter—are typically regarded as foreseeable and therefore do not break the chain of proximate causation.

108
Q

Late one night, a 25-year-old woman climbed over her next-door neighbor’s fence to get a look at a new deck that the neighbor had recently built. The neighbor was on vacation and unaware that the woman was on his property. After climbing over the neighbor’s fence, the woman tripped and fell on leftover lumber that the neighbor’s builder had left in some tall grass after building the deck. The neighbor was aware that the builder had placed the lumber in the grass, but the neighbor had placed no warning sign in the area. The woman broke her ankle and filed a negligence action against the neighbor seeking damages.

In a jurisdiction that follows the traditional approach regarding the duty owed to land entrants, does the woman have a valid claim for negligence against her neighbor?

Quote the rule implicated. Distinguish it with the modern approach.

Last time you didn’t know the rules.

A

No, because the woman was a trespasser.

Under the traditional approach, a land possessor generally owes no duty to trespassers—i.e., persons who intentionally enter another’s land without permission—unless the land possessor knows of or has reason to anticipate their presence.* A land possessor who breaches this duty and causes the plaintiff physical harm is liable for negligence.

Last time you chose: Yes, because the neighbor owes trespassers a duty to warn of hidden dangers. This is wrong because the woman was an undiscovered and unanticipated trespasser. Thus neighbor owed her no duty.

109
Q

The friend of a teacher owned a pumpkin patch that was not open to the public. The friend invited the teacher and his schoolchildren to enter the patch to pick their own pumpkins as part of a class trip. While the teacher was on the friend’s property, a wild snake bit him as he reached into the patch to pick a pumpkin. Nearby, the friend had posted a large sign that stated, “Danger: Snakes and rodents may be present in the patch - exercise caution when picking pumpkins.” The teacher brought a suit for negligence against the friend to recover damages for his injury.

The jurisdiction applies the traditional rules of landowner liability.

Is the friend likely to be held liable?

You need to know the rules!

A

No, because the friend warned the teacher of possible snakes.

Under the traditional common-law approach,* land possessors owe a duty of reasonable care to foreseeable land entrants—including persons who enter the land with permission (i.e., licensees). When this occurs, the land possessor owes licensees a duty to (1) warn about concealed dangers that are known or should be obvious to the land possessor and (2) use reasonable care in active operations conducted on the land. A land possessor who breaches this duty and causes the licensee physical harm is liable for negligence.

110
Q

A chef agreed to purchase a rare bottle of wine from a wine collector’s personal collection. The wine collector immediately shipped the bottle overnight to the chef through a delivery company that specialized in shipping wine. The next evening, the chef served the bottle of wine, but the wine had been spoiled, making him and his guests ill and rendering the wine worthless. The chef sued both the wine collector and the delivery company in negligence to recover damages.

At trial, the chef established that the bottle of wine had been subjected to high temperatures that were inappropriate for preserving wine. However, the chef was unable to establish which defendant had improperly stored the wine before it reached him. As a result, the chef did not present any evidence that allowed the jury to allocate the amount of damages between the two defendants.

The jurisdiction follows the doctrine of joint and several liability as well as the traditional standard for res ipsa loquitur.

Is the chef likely to succeed in his negligence suit?

Last time you had a lucky guess. Read the question carefully and have a clear understanding for the rules.

A

No, because the chef cannot prove that both of the defendants acted negligently.

If multiple defendants were negligent and any one of them could have caused the plaintiff’s harm, joint and several liability allows the plaintiff to recover even if it is impossible to prove which defendant actually caused the harm. But the plaintiff must first show that each of the defendants was negligent.

111
Q

A large oil company purchased a plot of land in a rural area and erected an oil well in compliance with local zoning codes. Many families living in the surrounding acres could hear the oil well operating loudly at all hours of the day and night. Although the noise from the oil well disturbed the entire neighborhood, only a small for-profit botanical garden nearby suffered economic loss as a result. For years, the garden had drawn most of its customers with a bird-watching hike during the migration period of a rare type of songbird. The persistent sounds of operations at the oil well drove the songbirds to avoid the garden during their migration. The noise from the oil well and the lack of songbirds caused the garden to lose most of its visitors, and it faced bankruptcy as a result.

If the garden brings a public nuisance claim against the oil company for damages for its economic loss, is the garden likely to succeed?

A

Yes, because the garden suffered harm that was different from the public at large.

A private plaintiff can sue for public nuisance—i.e., an unreasonable interference with a right common to the general public—only if the plaintiff sustained special damage different from that suffered by the public at large.

A plaintiff who suffers economic damages without any related personal injury or property damage cannot recover in a negligence action, but may recover in a public nuisance action.

112
Q

An on-duty firefighter responded to a 911 call from a tenant about a fire in his apartment. The fire was caused by the tenant when he fell asleep while smoking a cigarette in his bed. The firefighter who responded to the tenant’s call injured her ankle when she slipped and fell on a defective walkway outside the apartment building. The maintenance of the walkway was the responsibility of the apartment building owner, who knew the walkway needed to be repaired. Despite her ankle injury, the firefighter entered the tenant’s apartment to extinguish the fire, where she was further injured when a section of the apartment wall that was burned by the fire fell on her.

If the firefighter files an action for negligence against the tenant and the owner, which of the following best describes the likely outcome?

A

The firefighter can recover from the owner, but not the tenant.

Under the rescue doctrine, persons who negligently endanger themselves or others are liable for injuries sustained by rescuers. But the rescue doctrine is limited by the firefighter’s rule, which applies to all professional rescuers (e.g., firefighters, police officers). This rule bars professional rescuers from recovering for harm that resulted from the special dangers of their jobs—e.g., a firefighter extinguishing a fire.

Here, the firefighter was injured after she entered the tenant’s apartment to extinguish the fire and a section of the apartment wall that was burned by the fire fell on her. However, the firefighter’s rule bars recovery because the firefighter’s injury resulted from a special danger of her job. Therefore, the firefighter cannot recover from the tenant on her negligence claim

You changed your answer to the wrong choice. You chose: The firefighter cannot recover from the owner or the tenant.

113
Q

A pool cleaner at a country club used chlorine purchased from a company to kill the harmful bacteria that grew in his customers’ pools. The company sold its chlorine in plastic containers with plastic screw-tops. The containers had a warning that stated, “Caution: Chlorine,” but the warning did not specifically state that chlorine fumes were harmful if inhaled. The label also warned that the containers may leak if not stored upright. Despite the warning, the pool cleaner haphazardly stacked the containers on their sides and stored them in a small shed.

A maintenance worker assigned to organize the shed sustained injuries after inhaling excess fumes that had built up in the shed from the leaking containers. The maintenance worker filed suit against the company under a theory of strict products liability. At trial, the company established that a completely leak-free container for the chlorine was too expensive to manufacture and that an ordinary user of its product would be aware of the dangers of chlorine fumes, without an additional warning.

The jurisdiction applies the risk-utility test to determine whether a design defect exists.

Is the maintenance worker likely to succeed on his claim?

A

No, because there was no economically feasible alternative design for the containers.

Under the risk-utility test, a product is defectively designed if (1) the design creates a foreseeable risk of physical harm and (2) that risk could have been mitigated by a reasonable alternative design—e.g., a safer design available at a reasonable cost.

114
Q

Intentional misrepresentation

A

To establish a prima facie case of intentional misrepresentation (i.e., fraud or deceit), the plaintiff must show that:

  1. the defendant knowingly or recklessly misrepresented a material fact with the intent to induce the plaintiff’s reliance and
  2. the plaintiff reasonably relied on the misrepresentation and suffered pecuniary loss (i.e., monetary loss) as a result.
115
Q

The owner of an apartment complex employed a pest control company as an independent contractor to remove unwanted insects from one of the two buildings in the complex. The pest control company fumigated the building with a toxic gas. Even though the company exercised reasonable care, the gas escaped into the owner’s other building, which adjoined the fumigated building, where the gas caused serious illness to a tenant therein. The tenant had received a written advance notice about the fumigation that advised the tenant of the need to vacate his apartment during the hours the fumigation was conducted. The tenant had intended to leave his apartment, but he fell asleep just prior to the fumigation due to a medication he was taking for a medical condition.

The applicable jurisdiction treats fumigation as an abnormally dangerous activity and adheres to the rule of contributory negligence.

The tenant brought a strict liability action against the owner of the apartment complex to recover damages for his harm.

Who will likely prevail?

You got fooled by this question last time. You initially picked the right answer and then changed it to a wrong one.

A

The tenant, because the owner is vicariously liable for the harm that resulted from the fumigation.

A person who hires an independent contractor is generally not vicariously liable for torts committed by the independent contractor. However, vicarious liability will be imposed when the independent contractor’s work involved a nondelegable duty, such as when the independent contractor performed an abnormally dangerous activity.

Last time you chose: The owner, because the fumigation was conducted by an independent contractor.

116
Q

Recall the practice MBE question about the drug dog barking when it detected drugs during a traffic stop.

A

The rule against hearsay bars the admission of an out-of-court statement made by a person—not a machine or animal—that is offered to prove the truth of the matter asserted therein.

117
Q

A witness to an armed robbery identified a suspect in a proper police lineup that was not attended by the suspect’s attorney. Charges were brought against the suspect, but the witness, a tourist from out of the country, had returned to her home country before the trial began. At trial, the prosecutor seeks to introduce the witness’s prior statement of identification into evidence. The defendant objects to the introduction of the evidence.

Should the court allow the prior statement of identification into evidence? State the rule implicated.

You didn’t read this question carefully!

A

A declarant’s prior statement that identifies a person as someone the declarant perceived earlier is nonhearsay if the declarant testifies and is subject to cross-examination about the statement.

118
Q

A defendant is acquitted of murder. Subsequently, the family members of the victim bring a wrongful death action against the defendant. The defendant seeks to introduce a properly authenticated, certified copy of the final judgment to show that the defendant did not wrongfully kill the victim. The victim’s family members object to the introduction of the judgment.

May the defendant introduce the copy of the final judgment from his criminal case?

I still don’t understand the reasoning behind this question???

A

No, because the judgment is inadmissible hearsay.

Hearsay is an out-of-court statement (here, the judgment of acquittal) offered to prove the truth of the matter asserted therein (that the defendant did not wrongfully kill the victim). Hearsay is inadmissible absent an exclusion or exception. One exception applies to judgments of conviction

119
Q

When is a prior inconsistent statement admissible as nonhearsay, and when can it be introduced extrinsically for impeachment purposes?

A

A prior inconsistent statement is admissible nonhearsay if

  1. it was given under penalty of perjury at a trial, hearing, deposition, or other proceeding and
  2. the declarant testifies and is subject to cross-examination.

It can also be introduced extrinsically for impeachment purposes if the witness has the opportunity to explain or deny, and the adverse party can examine the witness about the statement.

120
Q

A father and son are charged with burglary. Prior to the trial, the prosecutor approached the son and asked him if he would be willing to testify against his father in exchange for a reduced sentence. After discussing the son’s role in the burglary and some negotiation, the prosecutor and the son’s defense attorney reached a plea agreement. The son pleaded guilty and was called to testify against his father. On cross-examination of the son, the father’s attorney brought up the fact that the son was also originally charged with the burglary and asked whether it was true that he received a lesser sentence for agreeing to testify against his father. The prosecutor objected to this line of questioning.

How should the court rule on the prosecutor’s objection?

A

The objection should be overruled, because the question concerns bias.

A witness can be impeached with evidence of self-interest or bias that may motivate the witness to testify falsely, such as benefits received in exchange for testimony.

121
Q

A defendant was charged with and tried for a crime. During the presentation of its case-in-chief, the prosecution introduced an inculpatory statement made by the defendant in an email regarding the commission of the crime. The defendant requested the immediate introduction of a subsequent related email sent by the defendant that contained an exculpatory statement. The defendant established that fairness requires that the two statements be considered at the same time.

Should the court honor the defendant’s request?

A

Yes, because fairness requires the two statements be considered at the same time.

Under the rule of completeness, a party may introduce any part of a previously admitted writing or recorded statement, or any other writing or recorded statement, that in fairness should be considered at the same time. This is true even if the additional evidence is otherwise inadmissible.

122
Q
A

Character evidence is admissible if a person’s character is an essential element of a civil claim, criminal charge, or asserted defense.

123
Q

A defendant was on trial for the attempted murder of his employer. The prosecutor called the defendant’s wife to the witness stand and asked her to recall anything the defendant said prior to leaving their home on the morning that the attempted murder took place. The wife sought to testify that while they were in bed, the defendant stated, “I’ve had enough of the way my employer treats me. Something has to be done.” The defendant objected to his wife’s proposed testimony. The defendant and his wife had legally separated after the defendant was arrested.

Is the wife’s testimony admissible?

You should know the difference. last time your chose an attractive wrong answer

A

The marital-communications privilege protects confidential communications made between spouses during the marriage. Under the majority view, either spouse may assert the privilege—even after termination of the marriage—and (1) refuse to testify about the communication or (2) prevent the other spouse from testifying.

Spousal-immunity
* Shields witness-spouse from testifying against spouse in criminal case about matters that occurred before or during marriage

  • privilege held by Witness-spouse
  • applies Only during active marriage

Marital-communications
* Protects against disclosure of confidential spousal communications made during marriage in criminal & civil cases

  • privilege held Both spouses
  • Continues after divorce
124
Q
A

Evidence of a person’s habit is admissible to prove that the person acted in accordance with that habit on a particular occasion.

125
Q
A
126
Q

An accountant and a banker were charged with conspiracy to commit embezzlement. The banker agreed to testify against the accountant in exchange for a plea bargain. However, at the accountant’s trial, the banker unexpectedly testified that she had embezzled the funds without the accountant’s knowledge or assistance. While the banker was still subject to being recalled as a witness, the prosecution called the banker’s secretary to testify that the banker said, “I gave the accountant what he needed to take the money, and no one will notice for months. He just wired me my share, so let’s go celebrate!”

Is the secretary’s testimony admissible?

A

Yes, because it is a prior inconsistent statement.

Here, the statement was made by the accountant’s coconspirator (the banker). But it was not made during the conspiracy since the embezzlement was complete and the banker had received her share. It also was not in furtherance of the conspiracy because it was not intended to help them perform their roles. Therefore, the banker’s statement is not excluded from hearsay as a statement by a coconspirator. And since the statement does not fall under any other hearsay exclusion or exception, it is not admissible substantively.

However, a prior inconsistent statement may be used to impeach a witness on a material issue even when it is not admissible substantively. A party may impeach the witness with the statement by:

examining the witness about the statement or

introducing the statement through extrinsic evidence if (1) the witness has an opportunity to explain or deny—and the opposing party has the opportunity to question the witness about—the statement or (2) justice so requires.

A prior inconsistent statement may be used to impeach a witness on a material issue even when it is not admissible substantively.

127
Q

A plaintiff sued a defendant under a disabilities discrimination statute, alleging that the defendant refused to hire the plaintiff because of her physical disability. The defendant has asserted that he refused to employ the plaintiff because he reasonably believed that she would be unable to perform the job. The defendant sought to testify that the plaintiff’s former employer advised him not to hire the plaintiff because she was unable to work productively for more than three hours each day.

Is the defendant’s testimony admissible?

A

Yes, as evidence of the defendant’s reason for refusing to hire the plaintiff.

Here, evidence pertinent to the defendant’s hiring decision is relevant in this discrimination suit. The defendant testified that he did not hire the plaintiff because he reasonably believed that she was unable to perform the job. That belief stemmed from the former employer’s out-of-court statement that the plaintiff was unable to work productively for more than three hours a day. Although this statement cannot be used to prove the truth asserted therein, it is admissible to show the statement’s effect on the defendant’s hiring decision

The hearsay rule does not bar out-of-court statements that are offered for a purpose other than to prove the truth of the matter asserted.—e.g., to show the statement’s effect on the listener.

128
Q

The defendant’s attorney in a fraud case called a witness to testify as to the defendant’s character. On cross-examination, the prosecutor asked the witness whether he had ever been arrested for writing bad checks. In fact, the witness had been arrested two years ago for writing bad checks, but the charges had been dropped due to a lack of evidence that the witness had committed the crime. The defendant’s attorney objects to the question.

Should the prosecutor be allowed to ask the question?

A

No, because the witness was never convicted of the crime.

A party can attack any witness’s character for truthfulness with reputation or opinion testimony OR with specific instances of conduct (SICs) that are probative of that character (Choice A). Only two types of SICs are admissible for this purpose: (1) convictions for a felony or crime of dishonesty and (2) other bad acts. However, a mere arrest does not qualify as a bad act that can be used to attack a witness’s character for truthfulness. That is because an arrest for misconduct is not itself misconduct.

129
Q

Admissibility of out-of-court statements

A
130
Q

A defendant is on trial for embezzling $50,000 from his former employer. The prosecution wishes to offer into evidence an anonymous letter, in its entirety, that was received by the defendant’s former supervisor. The supervisor testified that the letter was written in the defendant’s handwriting, which the supervisor knew from their years of working together. The letter reads, “I am consumed by guilt for what I have done. Here is half the money I took from you, and if you promise not to prosecute, I will send you the rest later this year. If you accept this arrangement, please post a personal ad in the local paper using the phrase ‘All is forgiven.’” The note was accompanied by $25,000 in cash. The defense objects to the admission of the letter.

Is the letter admissible?

A

Yes, because the letter contains statements by an opposing party to the current litigation.

Certain statements are excluded from the rule against hearsay and are therefore considered nonhearsay. One exclusion applies to statements made by and offered against an opposing party.

131
Q

Attorney-Client Privilege

A

The attorney-client privilege protects communications between an attorney and client that were (1) made to obtain legal advice or representation and (2) intended to be and kept confidential. But this privilege does not apply to communications in which a client seeks advice in furtherance of an ongoing or future crime or fraud.

132
Q

In a diversity action brought in federal court, a plaintiff seeks to establish her claim to the proceeds from her husband’s life insurance policy by showing that her husband, who has been absent for eight years, is dead. State law in the jurisdiction where the court is located provides that an irrebuttable presumption arises that a person is dead when a party establishes that the person has been missing and not heard from for more than seven years.

What law should the court use to determine the effect the presumption has on the plaintiff’s claim?

A

The state law, because it supplies the rule of decision in the case.

Federal Rule of Evidence 302 governs the application of law to presumptions in federal civil cases. Under this rule, when state law supplies the rule of decision for a claim or defense (i.e., in diversity cases such as this one), the court should apply state law to determine the effect of a presumption on the claim or defense. Therefore, the court should apply the state law regarding the effect of the presumption of death.

133
Q

A convenience store clerk was complaining about his financial troubles to his best friend. The friend said that the clerk’s employer had been cheating the clerk out of a decent salary for too many years, and that the employer owed the clerk. The friend suggested that if the clerk robbed the store during another clerk’s shift, he would never get caught. The friend offered to loan the clerk his gun to use to scare the clerk on duty. Both men agreed that no one would get hurt in the process. The next day, the clerk carried out the plan to rob the store while the friend waited outside in a car. During the robbery, the clerk accidentally discharged the gun, and a customer was shot and died instantly. The clerk panicked and left the store empty-handed. The friend drove the clerk back to his mother’s house, told him to lay low, and then drove home. The clerk later decided that he needed to get out of town quickly. He stole his mother’s car, which was more reliable than his own, to drive to a nearby state.

In a case against the friend, which of the following charges would most likely be successfully prosecuted?

A

Felony murder.

The friend, as an accomplice, is liable for crimes committed by the clerk that were a natural and probable consequence of the encouraged robbery, which includes felony murder. Therefore, the friend could be successfully prosecuted for felony murder—but not attempted robbery since it would merge into felony murder

Felony murder is an unintended killing proximately caused by and that occurs during the commission or attempted commission of an inherently dangerous felony (e.g., robbery). The underlying felony, as the lesser included offense, typically merges into the felony-murder charge.

Inherently dangerous felonies
(basis for felony murder)

Burglary
Arson
Rape
Robbery
Kidnapping
Mnemonic: BARRK

134
Q
A
135
Q

The defendant, his brother, and his best friend formed a plan to rob a bank. On the day of the crime, the defendant and his brother entered the bank carrying guns, while the best friend stayed in the car to act as a getaway driver. After the defendant received one bag of money from a bank teller, he saw a security guard pull out a gun. The defendant tried to shoot the security guard but instead shot his brother. The defendant panicked and ran out of the building toward the getaway car. The security guard chased the defendant and fired a shot toward him as he approached the car. The shot hit the best friend, who was in the driver’s seat of the car. Shortly thereafter, the police arrived and arrested the defendant. The brother and the best friend later died as a result of their gunshot wounds.

For which of the following crimes is the defendant most likely to be convicted and punished?

A

One count of felony murder for death of the brother.

Felony murder is the unintended killing of another proximately caused by and during the commission or attempted commission of an inherently dangerous felony. The death of a cofelon (i.e., an accomplice) can support a felony-murder conviction when the defendant unintentionally kills the cofelon while committing or attempting to commit the inherently dangerous felony.

In most jurisdictions, the underlying felony (here, robbery) merges into felony murder. This means that a defendant convicted of felony murder cannot also be convicted of the underlying felony. Therefore, the defendant cannot be properly convicted of robbery.

Inherently dangerous felonies (basis for felony murder)

Burglary
*Arson
*Rape
*Robbery
*Kidnapping

Mnemonic: BARRK

136
Q
A

Robbery is larceny from the victim’s person or presence by force (battery) or intimidation (assault). Since larceny and battery/assault are lesser included offenses to robbery, they merge into the completed robbery. Therefore, the defendant may only be convicted of robbery—not its constituent offenses.

137
Q
A

Common-law arson always requires fire damage to the actual building structure—not just the contents of the dwelling. And for common-law burglary, under any approach, the requisite intent must exist at the time of entry.

138
Q
A

Larceny by trick is a larceny accomplished by fraud or deceit that results in the conversion of the property of another. Larceny by trick differs from false pretenses in that the defendant acquires mere possession of (not title to) the property.

139
Q

A woman broke into her ex-husband’s house late one night when she knew he was away on business, intending to take a sculpture that he had been awarded in their divorce settlement. She searched the entire house but was unable to find the sculpture. She figured he had probably sold the sculpture, which made her furious because he knew how much she loved it. In a rage, she slashed a painting of his new girlfriend before leaving the house.

Of which of the following crimes is the woman guilty?

A

Burglary and attempted larceny.

Common law burglary* requires proof of the following:

an unlawful breaking and entering of another’s dwelling at night and the specific intent to commit a felony (e.g., larceny) therein.

Once the defendant enters the dwelling with the requisite intent, the burglary is complete. This means that there is no need to prove that the defendant completed the underlying felony to obtain a burglary conviction.

A burglary defendant who did not complete the underlying felony is also guilty of the attempted commission of that felony. That is because the defendant (1) specifically intended to commit the felony and (2) performed an overt act in furtherance of the felony—the unlawful breaking and entering.

140
Q
A

Conspiracy is (1) an agreement between two or more persons to accomplish an unlawful purpose (2) with the specific intent to accomplish that purpose. The common law follows the bilateral approach to conspiracy, which requires proof of at least two guilty minds. As a result, a defendant cannot be convicted of this crime if the other alleged conspirator(s) merely feigned agreement.

141
Q

A gang member decided to rob a bank and approached a fellow member of his gang to invite him to participate. The other member, who was actually an undercover officer, agreed. The gang member purchased a toy gun, planning to use it to threaten the bank teller during the robbery. When the gang member went to meet the undercover officer the next day behind the bank they had planned to rob, he was immediately arrested. The applicable jurisdiction has adopted the Model Penal Code’s definition of conspiracy.

Can the gang member be convicted of conspiracy?

A

Yes, because the undercover officer’s false agreement does not negate any required element of the crime because the MPC requires only one guilty mind.

Had this jurisdiction adopted the common-law, bilateral approach to conspiracy, the undercover officer’s feigned agreement would have prevented the gang member from being convicted of conspiracy. That is because the bilateral approach requires proof of at least two guilty minds.

142
Q

A man planned to rob a city bank. Waiting until he saw the bank’s security guard leave the building to take a walk around the perimeter, the man entered the bank and walked up to a teller with his hand pointed in his pocket, as if he had a gun. The teller had her back to the man and did not see him at all. Before the teller could turn around, the security guard reentered the building. Seeing the guard, the man took his hand out of his pocket and ran out of the building. The man was arrested and charged with attempted robbery of the bank.

Should the man be found guilty?

A

Yes, whether or not he abandoned his plan.

In most jurisdictions, the crime of attempt is completed once the defendant has taken a substantial step by engaging in conduct that exceeds mere preparation and tends to effect the commission of the crime. As a result, a defendant who has taken a substantial step may not legally abandon (or withdraw from) the attempt due to a change of heart or fear of apprehension.*

Here, the man possessed the specific intent to rob the bank when he entered the building, pretending to have a gun in his pocket. This entry constituted a substantial step toward the commission of the bank robbery and completed the attempt. As a result, the man could not abandon or withdraw from the attempt by running out of the bank and should be found guilty of attempted robbery.

143
Q

Here, the daughter took a substantial step toward murdering her parents when she put what she believed to be poisonous castor oil seeds in their oatmeal with the specific intent to kill them. However, she could not have killed them because she had unknowingly used the innocuous seeds from the false castor oil plant. But since factual impossibility is never a defense to attempt, she can still be found guilty of attempted murder.

name the rule

A

Attempt occurs when a person (1) takes a substantial step toward the commission of a crime (2) with the specific intent to commit the crime. Factual impossibility is never a defense to attempt.

144
Q
A

A defendant commits murder when he/she unlawfully kills another with malice aforethought. However, the defendant should not be convicted of this crime if the murder was justified because the defendant was acting in self-defense.

If the defendant was not the aggressor, then the defendant may use reasonble force against another to prevent immediate unlawful harm. Since the defendant can use only as much force as is required to repel the attack, deadly force can be used only when it is reasonably necessary to prevent (1) death or serious injury or (2) the commission of a serious felony involving a risk to human life.

Justification means the crrime’s positive effect outweighs negative effcts

145
Q
A

Larceny is the trespassory taking and carrying away of another’s personal property with the specific intent to permanently deprive the person of that property. It is no defense that the defendant later returned the property to the place from which it was taken or to its rightful owner.

146
Q
A

Elements of larceny

  • Trespassory – without owner’s consent
  • Taking – removal from owner’s possession
  • Carrying away – movement or asportation of property, however slight
  • Personal property – anything subject to ownership except real property, intangibles & services
  • Another’s property – property must be in possession of someone other than defendant
  • Intent to permanently deprive – specific intent to steal must exist at time property is taken
147
Q

Two women and a man were arrested and charged with conspiracy to rob a bank. No one else was involved. The three were tried before a jury in a single proceeding. The two women were acquitted of the conspiracy, but the man was found guilty.

In a jurisdiction that follows the common-law approach to conspiracy, should the man’s conviction be overturned on appeal?

You didn’t read carefully!

A

Yes, because the three defendants were tried in a single proceeding.

At common law, a conspirator cannot be convicted of conspiracy if all other coconspirators are acquitted at the same trial.

You chose the wrong answer choice: No, because of the application of the Pinkerton Rule.

The Pinkerton Rule holds all conspirators liable for any foreseeable crimes committed by a coconspirator in furtherance of the conspiracy—regardless of whether they had knowledge of those crimes.

148
Q

An employee was up for a promotion but was passed over by his boss for a female colleague with more experience. After learning that he had not received the promotion, the employee became angry with his boss and convinced himself that the colleague and the boss, who was married, were involved in a relationship. He thereafter contacted the boss’s wife and convinced her to murder the boss. The employee and the wife stated that they would not harm the colleague, as it might make their involvement too obvious. The employee provided the wife with a gun.

The next day, the wife approached the colleague and the boss in their office parking lot. The wife, who was not an experienced shooter, shot at the boss but hit the colleague in the arm, panicked, and then ran off. An onlooker rushed the colleague to the hospital. Although the injury was not life threatening, the colleague contracted an infection during surgery and died the following week. A later investigation revealed that the infection was a result of medical malpractice that occurred during the surgery. The crime was eventually traced back to the employee and the wife, and they were both charged in connection with the shooting.

The employee is most likely to be convicted of which of the following crimes?

Read a little more carefully

A

Murder, attempted murder, and conspiracy to commit murder.

Here, the colleague would not have died but for the wife’s conduct (actual causation), and the colleague’s death was a foreseeable consequence of that conduct (proximate causation)—notwithstanding the medical malpractice. And since the killing was committed in furtherance of the conspiracy, the wife and the employee—as her coconspirator—are also guilty of murder of the colleague

Under the common law, no overt act is required.

The crimes of attempted murder and murder do not merge here because there were two different victims—the boss and the colleague. If the wife had shot and killed the boss—i.e., successfully completed the intended crime—instead of shooting the colleague, then attempted murder and murder would have merged.

149
Q

An off-duty police officer was having a drink with his wife in a bar. A man walked by their table and made a lewd comment about the wife as he passed. The officer jumped to his feet and asked the man to repeat what he had said. The man, who was obviously intoxicated, repeated the comment. The officer then pushed the man, causing the man to fall backward. The man rose to his feet, pulled a knife from his waistband, and walked toward the officer. As the man lunged at the officer with the knife, the officer pulled his gun from his waistband and shot the man in the leg. Although the officer did not intend to fire a lethal shot, the man had a disease that prevented his blood from properly clotting, and he died from blood loss due to the injury.

Is the officer likely to be convicted of murder?

A

No, because the officer was entitled to use self-defense under the circumstances.

An initial aggressor can claim self-defense only if (1) the aggressor’s use of nondeadly force was met with deadly force or (2) the aggressor, in good faith, completely withdrew from the altercation and communicated that fact to the victim.

150
Q
A